[obm-l] Uma recorrência diferente

2023-04-05 Por tôpico Professor Vanderlei Nemitz
Oi, mestres!

Estava resolvendo um problema de combinatória e obtive essa recorrência:

*x(n) = x(n - 1) + (n - 1).x(n - 2), com x1 = 1 e x2 = 2*.

Por exemplo, x3 = x2 + 2.x1 e x9 = x8 + 8.x7

Como resolver quando os coeficientes não são todos constantes?

Apenas como curiosidade, o problema que originou a recorrência é:
(IME - RJ) - Um professor dá um teste surpresa para uma turma de 9 alunos,
e diz que o teste pode ser feito sozinho ou em grupos de 2 alunos. De
quantas formas a turma pode ser organizar para fazer o teste? (Por exemplo,
uma turma de 3 alunos pode ser organizar de 4 formas e uma turma de 4
alunos pode se organizar de 10 formas)



Não
contém vírus.www.avast.com

<#DAB4FAD8-2DD7-40BB-A1B8-4E2AA1F9FDF2>

-- 
Esta mensagem foi verificada pelo sistema de antiv�rus e
 acredita-se estar livre de perigo.



[obm-l] Re: Questão de probabilidade

2022-03-16 Por tôpico Professor Vanderlei Nemitz
Hummm...
Acho que descobri o que o autor pensou.
Parece que 6 casos, distribuindo P, P, B, B, B, B, M (2 pretos, 4 brancos e
1 malhado) entre 3 gatos e 4 cachorros.
Mas...esse espaco amostral é equiprovável???

Em qua., 16 de mar. de 2022 07:58, Professor Vanderlei Nemitz <
vanderma...@gmail.com> escreveu:

> Bom dia!
> Na questão a seguir, do vestibular da UEM, penso que o espaço amostral tem
> 105 elementos, pois um cachorro é preto (desconsideramos esse). Porém, com
> esse pensamento, não consigo obter o gabarito, que diz que 02 e 16 são
> corretas.
> Alguém poderia ajudar?
> Muito obrigado!
>
> *Em um pet shop há 3 gatos e 5 cães. Sabemos que 3 desses animais são
> pretos, 4 são brancos e 1 é malhado. Além disso, pelo menos 1 cachorro é
> preto. Assinale o que for correto. *
> *01) A probabilidade de haver exatamente 1 cachorro preto é de 1/6. *
> *02) A probabilidade de haver pelo menos 1 gato branco e pelo menos 2
> cachorros brancos é de 2/3.*
> *04) A probabilidade de haver um cachorro malhado é maior do que a
> probabilidade de haver um gato malhado. *
> *08) Se um animal for escolhido ao acaso, a probabilidade de ele ser um
> cachorro preto é de 1/8. *
> *16) Se um animal for escolhido ao acaso, a probabilidade de ele ser um
> gato malhado é de 1/16.   *
>

-- 
Esta mensagem foi verificada pelo sistema de antiv�rus e
 acredita-se estar livre de perigo.



[obm-l] Questão de probabilidade

2022-03-16 Por tôpico Professor Vanderlei Nemitz
Bom dia!
Na questão a seguir, do vestibular da UEM, penso que o espaço amostral tem
105 elementos, pois um cachorro é preto (desconsideramos esse). Porém, com
esse pensamento, não consigo obter o gabarito, que diz que 02 e 16 são
corretas.
Alguém poderia ajudar?
Muito obrigado!

*Em um pet shop há 3 gatos e 5 cães. Sabemos que 3 desses animais são
pretos, 4 são brancos e 1 é malhado. Além disso, pelo menos 1 cachorro é
preto. Assinale o que for correto. *
*01) A probabilidade de haver exatamente 1 cachorro preto é de 1/6. *
*02) A probabilidade de haver pelo menos 1 gato branco e pelo menos 2
cachorros brancos é de 2/3.*
*04) A probabilidade de haver um cachorro malhado é maior do que a
probabilidade de haver um gato malhado. *
*08) Se um animal for escolhido ao acaso, a probabilidade de ele ser um
cachorro preto é de 1/8. *
*16) Se um animal for escolhido ao acaso, a probabilidade de ele ser um
gato malhado é de 1/16.   *

-- 
Esta mensagem foi verificada pelo sistema de antiv�rus e
 acredita-se estar livre de perigo.



Re: [obm-l] Probabilidade

2021-04-08 Por tôpico Professor Vanderlei Nemitz
Muito legal esse tipo de problema.
Em que ano caiu, você sabe, Pacini?

Em sáb., 3 de abr. de 2021 às 15:22, Pacini Bores 
escreveu:

> Olá pessoal, Encontrei uma resposta que não está entre as opções desta
> questão do Canguru.
>
> " um certo jogo tem um vencedor quando este atinge 3 pontos a frente do
> oponente. Dois jogadores A e B estão jogando e, num determinado momento, A
> está 1 ponto a frente de B. Os jogadores  têm probabilidades iguais de
> obter 1 ponto. Qual a probabilidade de A vencer o jogo ?
>
> (A) 1/2   (B) 2/3  (C) 3/4   (D) 4/5  (E) 5/6
>
>
>
> O que vocês acham ?
>
>  Pacini
>
>
> --
> Esta mensagem foi verificada pelo sistema de antivírus e
> acredita-se estar livre de perigo.
>

-- 
Esta mensagem foi verificada pelo sistema de antiv�rus e
 acredita-se estar livre de perigo.



[obm-l] Álgebra

2021-01-28 Por tôpico Professor Vanderlei Nemitz
Oi, pessoal, tudo bem?
Tentei algumas coisas nesse problema, enxergar a, b, c, d como senos e
cossenos ou utilizar números complexos, mas não obtive êxito.
A resposta é 1.
Para casos particulares é fácil chegar nesse valor.

Se alguém resolver, agradeço muito!

a/b + c/d = –1
a^2 + c^2 = 1
b^2 + d^2 = 1
Calcule b^3/a + d^3/c.

Imagino que a, b, c, d são reais, certo? Nada é dito...


Re: [obm-l] Amigo secreto ENEM

2021-01-28 Por tôpico Professor Vanderlei Nemitz
Muito obrigado a todos pelas mensagens.
Como a gente aprende por aqui!!!

No fim das contas a questão foi anulada pelo INEP.
Como disse o Claudio Buffara, daria um ótimo artigo!


Em qui., 28 de jan. de 2021 às 11:38, Arthur Queiroz 
escreveu:

> Uma pergunta: você assume que o número de sorteios é !10. Mas e se, em
> meio ao sorteio, nossa permutação caótica seja tal que seja formado um
> ciclo indesejado? Digamos A->B->C->A. Como o sorteio continuará nesse caso?
> Será escolhida aleatoriamente uma pessoa de fora do ciclo para continuar?
> Isso não afetaria esse !10?
>
> Em ter, 26 de jan de 2021 17:26, Ralph Costa Teixeira 
> escreveu:
>
>> Deixa eu copiar o que escrevi em outro lugar... :D :D
>>
>> Primeiro: não fica claro do enunciado se "auto-sorteios" (alguém sortear
>> o próprio nome) são permitidos ou não, e isto ALTERA a resposta. :(
>>
>> Vejamos possíveis respostas corretas:
>>
>> ---///---
>>
>> SE AUTO-SORTEIOS FOREM PERMITIDOS:
>> Em resumo, temos 1/10 de chance de A iniciar o sorteio, e 1/10 de chance
>> de B terminar (1/10 sim, pois A *pode* terminar). Assim, a resposta seria
>> 1/10*1/10*2=1/50.
>>
>> Com mais detalhes para justificar o segundo "1/10":
>> -- Número de sorteios possíveis = 10!
>> -- Número de sorteios que formam um único ciclo de tamanho 10 = 9!
>> -- Note que ter um ciclo de tamanho 10 equivale a terminar com quem
>> inicia; portanto, a chance de o amigo secreto terminar com o mesmo que
>> iniciou seria 9!/10!=1/10 (*que é independente de quem começa*).
>>
>> Assim:
>> -- Chance de A iniciar = 1/10;
>> Agora, DADO QUE A INICIOU:
>>  Chance de A terminar = 9!/10! = 1/10
>>  Portanto, chance de não terminar com A: 9/10
>>  Chance de B terminar (por simetria): (9/10) /9 = 1/10
>>
>> Isso nos dá 1/10*1/10 = 1/100 de chance do amigo secreto começar por A e
>> terminar com B. Portanto a resposta seria o dobro, 1/50.
>>
>> ---///---
>> SE AUTO-SORTEIOS SÃO PROIBIDOS:
>> -- Número de sorteios (desarranjos) possíveis = !10 (vou escrever K=!10
>> daqui por diante);
>> -- Número de sorteios que formam um único ciclo de tamanho 10 = 9!
>> -- Portanto, a chance de o amigo secreto terminar com o mesmo que iniciou
>> seria 9!/K (que é independente de quem começa).
>>
>> Assim:
>> -- Chance de A iniciar = 1/10;
>> Agora, DADO QUE A INICIOU:
>>  Chance de A terminar = 9!/K
>>  Portanto, chance de não terminar com A: 1-9!/K
>>  Chance de B terminar (por simetria): (1-9!/K) /9 = (K-9!)/(9K)
>>
>> Isso nos dá 1/10* (K-9!)/(9K) = (K-9!)/(90K) de chance do amigo secreto
>> começar por A e terminar com B. Portanto a resposta seria o dobro,
>> (K-9!)/(45K). Fazendo a conta com a ajuda do computador, achei 12001/741645.
>>
>> Abraço, Ralph.
>>
>>
>>
>> On Tue, Jan 26, 2021 at 1:45 PM Professor Vanderlei Nemitz <
>> vanderma...@gmail.com> wrote:
>>
>>> Oi, pessoal!
>>>
>>> Com certeza vocês estão acompanhando desde domingo as resoluções da
>>> questão do ENEM do amigo secreto.
>>> Além da resposta proposta, *1/45*, que parece não estar correta, já vi
>>> outras duas, *12001/741645* (ETAPA e ANGLO), que consideram também que
>>> o sorteio anterior para definir "quem presenteia quem", e *7/360*, do
>>> vídeo a seguir:
>>>
>>> https://www.youtube.com/watch?v=c-t_BAMASKE
>>>
>>> Gostaria da opinião (e se possível, uma resolução) dos especialistas da
>>> lista (Ralph e cia :))
>>>
>>> Muito obrigado!
>>>
>>>
>>>
>>>
>>>
>>


[obm-l] Amigo secreto ENEM

2021-01-26 Por tôpico Professor Vanderlei Nemitz
Oi, pessoal!

Com certeza vocês estão acompanhando desde domingo as resoluções da questão
do ENEM do amigo secreto.
Além da resposta proposta, *1/45*, que parece não estar correta, já vi
outras duas, *12001/741645* (ETAPA e ANGLO), que consideram também que o
sorteio anterior para definir "quem presenteia quem", e *7/360*, do vídeo a
seguir:

https://www.youtube.com/watch?v=c-t_BAMASKE

Gostaria da opinião (e se possível, uma resolução) dos especialistas da
lista (Ralph e cia :))

Muito obrigado!


[obm-l] Ângulos de um triângulo

2020-11-30 Por tôpico Professor Vanderlei Nemitz
Boa noite!
Alguém conhece uma saída para o seguinte problema?
Muito obrigado!

*Num triângulo isósceles ABC, AB = AC.*
*Seja D um ponto interno tal que os ângulos DBC, DCB, DBA e DCA medem,
respectivamente, 12°, 18°, 54° e 48°. *
*Determine a medida do ângulo DAC.*


Livre
de vírus. www.avast.com
.
<#DAB4FAD8-2DD7-40BB-A1B8-4E2AA1F9FDF2>


[obm-l] Polinômio

2020-10-25 Por tôpico Professor Vanderlei Nemitz
Bom dia!
Alguém tem uma saída interessante para esse problema?

Sejam r1, r2, ..., r20 as raízes do polinômio p(x) = x^20 - 7x^3 + 1. Se o
somatório de 1/[(rk)^2 + 1], com k variando de 1 a 20, é da forma m/n, com
m e n inteiros positivos e primos entre si, calcule m + n.

Espero ter escrito de forma clara o enunciado :)

Muito obrigado!


[obm-l] Resto da divisão de um polinômio

2020-08-22 Por tôpico Professor Vanderlei Nemitz
Oi!

Existe algum fato específico que ajude a determinar o resto da divisão de
um polinômio de grau elevado por outro, ou depende do caso?

Por exemplo, como encontrar o seguinte resto, sem excessivos cálculos?
Muito obrigado!

*Determine o resto da divisão do polinômio x^30 - x^28 + 7x^12 por x^2 +
x + 1?*


Livre
de vírus. www.avast.com
.
<#DAB4FAD8-2DD7-40BB-A1B8-4E2AA1F9FDF2>

-- 
Esta mensagem foi verificada pelo sistema de antiv�rus e
 acredita-se estar livre de perigo.



[obm-l] Re: [obm-l] Re: [obm-l] Elipse e lugar geométrico

2020-08-22 Por tôpico Professor Vanderlei Nemitz
Demorei para responder, mas queria dizer que foi muito boa sua resolução,
como sempre, Ralph!
Eu desconhecia o fato de as coordenadas do incentro serem dadas daquela
forma.

Muitíssimo obrigado!

Vanderlei

<https://www.avast.com/sig-email?utm_medium=email_source=link_campaign=sig-email_content=webmail>
Livre
de vírus. www.avast.com
<https://www.avast.com/sig-email?utm_medium=email_source=link_campaign=sig-email_content=webmail>.
<#DAB4FAD8-2DD7-40BB-A1B8-4E2AA1F9FDF2>

Em qui., 20 de ago. de 2020 às 00:37, Ralph Costa Teixeira <
ralp...@gmail.com> escreveu:

> As coordenadas do incentro sao a media ponderada das coordenadas dos
> vertices, usando os lados como pesos. Ou seja, se escrevo P=(5cost,4sint),
> F1=(-3,0), F2=(3,0) e Incentro=(x,y):
>
> x = (30cost + (-3)b + 3c) / 16
> y = (24sint + 0 + 0) / 16
>
> onde b=d(P,F2) e c=d(P,F1). Note que b+c=eixo maior = 10.
>
> Mais especificamente:
>
> b^2=d(P,F2)^2=(5cost-3)^2+(4sint)^2=9(cost)^2-30cost+25=(3cost-5)^2, ou
> seja, b=5-3cost, portanto c=5+3cost.
>
> Jogando na formula de x e y:
>
> x= 3cost ; y=3sint/2. Outra elipse, a saber, (x^2)/9+(y^2)/(9/4)=1 (talvez
> tirando os pontos onde tudo degenera, para ser chato).
>
> Abraço, Ralph.
>
>
>
> Hmm Assim:
>
> On Wed, Aug 19, 2020 at 11:58 PM Professor Vanderlei Nemitz <
> vanderma...@gmail.com> wrote:
>
>> Oi!
>> Venho com mais uma envolvendo incentro.
>>
>> *O ponto P pertence a uma elipse de focos F1 e F2 e de equação (x^2)/25 +
>> (y^2)/16 = 1. Determine o lugar geométrico do incentro do triângulo PF1F2.*
>>
>> Muito obrigado!
>>
>>
>> <https://www.avast.com/sig-email?utm_medium=email_source=link_campaign=sig-email_content=webmail>
>>  Livre
>> de vírus. www.avast.com
>> <https://www.avast.com/sig-email?utm_medium=email_source=link_campaign=sig-email_content=webmail>.
>>
>> <#m_6041639077674691514_m_2344932968934913062_DAB4FAD8-2DD7-40BB-A1B8-4E2AA1F9FDF2>
>>
>> --
>> Esta mensagem foi verificada pelo sistema de antivírus e
>> acredita-se estar livre de perigo.
>
>
> --
> Esta mensagem foi verificada pelo sistema de antivírus e
> acredita-se estar livre de perigo.

-- 
Esta mensagem foi verificada pelo sistema de antiv�rus e
 acredita-se estar livre de perigo.



[obm-l] Elipse e lugar geométrico

2020-08-19 Por tôpico Professor Vanderlei Nemitz
Oi!
Venho com mais uma envolvendo incentro.

*O ponto P pertence a uma elipse de focos F1 e F2 e de equação (x^2)/25 +
(y^2)/16 = 1. Determine o lugar geométrico do incentro do triângulo PF1F2.*

Muito obrigado!


Livre
de vírus. www.avast.com
.
<#DAB4FAD8-2DD7-40BB-A1B8-4E2AA1F9FDF2>

-- 
Esta mensagem foi verificada pelo sistema de antiv�rus e
 acredita-se estar livre de perigo.



[obm-l] Re: [obm-l] Re: [obm-l] Geometria plana com desigualdade de médias?

2020-08-17 Por tôpico Professor Vanderlei Nemitz
Muito obrigado, Matheus!
Pensei nas outras desigualdades, menos em Cauchy-Schwarz.

Muito bom!

Em dom, 16 de ago de 2020 10:11, Matheus Secco 
escreveu:

> Olá, Vanderlei.
> Por Cauchy-Schwarz, temos
>
> (a/ha + b/hb + c/hc) * (a*ha + b*hb + c*hc) >= (a+b+c)^2.  (#)
>
> Como (a*ha + b*hb + c*hc) = 2S, onde S é a área de ABC, segue que a
> expressão a/ha + b/hb + c/hc é pelo menos 2p^2/S, onde p é o
> semi-perimetro.
>
> Por outro lado, a igualdade em (#) ocorre se, e somente se, ha = hb = hc,
> ou seja, quando P é o incentro do triângulo
>
> Abraços,
> Matheus
>
> Em dom, 16 de ago de 2020 08:59, Professor Vanderlei Nemitz <
> vanderma...@gmail.com> escreveu:
>
>> Bom dia!
>>
>> Tentei utilizar alguma desigualdade de médias aqui, mas não tive êxito.
>> Alguém ajuda?
>> Muito agradecido!
>>
>> Seja P um ponto no interior de um triângulo e sejam ha, hb e hc as
>> distâncias de P aos lados a, b e c, respectivamente. Mostre que o valor
>> mínimo de (a/ha) + (b/hb) + (c/hc) ocorre quando P é o incentivo do
>> triângulo ABC.
>>
>> --
>> Esta mensagem foi verificada pelo sistema de antivírus e
>> acredita-se estar livre de perigo.
>
>
> --
> Esta mensagem foi verificada pelo sistema de antivírus e
> acredita-se estar livre de perigo.

-- 
Esta mensagem foi verificada pelo sistema de antiv�rus e
 acredita-se estar livre de perigo.



[obm-l] Geometria plana com desigualdade de médias?

2020-08-16 Por tôpico Professor Vanderlei Nemitz
Bom dia!

Tentei utilizar alguma desigualdade de médias aqui, mas não tive êxito.
Alguém ajuda?
Muito agradecido!

Seja P um ponto no interior de um triângulo e sejam ha, hb e hc as
distâncias de P aos lados a, b e c, respectivamente. Mostre que o valor
mínimo de (a/ha) + (b/hb) + (c/hc) ocorre quando P é o incentivo do
triângulo ABC.

-- 
Esta mensagem foi verificada pelo sistema de antiv�rus e
 acredita-se estar livre de perigo.



Re: [obm-l] Desafio de probabilidade

2020-07-25 Por tôpico Professor Vanderlei Nemitz
Então meu raciocínio foi muito errado, pois pensei assim:
Seja p a probabilidade de Zé Roberto vender. Podemos considerar que o jogo
"começa" com Zé Roberto precisando obter um 6 para vencer.
Assim, a probabilidade de Humberto vencer é:
q = (3/6).(1/6).p, ou seja, p = 12q
Assim, p = 12/13 e q = 1/13

Prezado Cláudio, você pode explicar sua resolução?

Muito obrigado!






Em sáb., 25 de jul. de 2020 às 13:43, Claudio Buffara <
claudio.buff...@gmail.com> escreveu:

> Eu achei 5/7.
>
> On Sat, Jul 25, 2020 at 7:28 AM Professor Vanderlei Nemitz <
> vanderma...@gmail.com> wrote:
>
>> Bom dia!
>> O problema a seguir encontra-se em uma prova de desafios da PUC-RJ, muito
>> boas!!!
>> Acho que são organizadas pelo professor Nicolau Saldanha.
>> Encontrei uma resposta bem alta, mais de 90%. Será que está correto?
>> Muito obrigado!
>>
>> Zé Roberto e Umberto gostam de jogar par ou ímpar; Zé Roberto sempre pede
>> par e Umberto sempre pede íımpar. Eles gostam de inventar novas maneiras de
>> jogar. A última maneira que eles inventaram usa um dado comum, com seis
>> faces numeradas de 1 a 6. Eles jogam o dado várias vezes até que um número
>> saia três vezes seguidas; Zé Roberto ganha se este número for par, Umberto
>> ganha se for ímpar. Sábado de manhã o dado teve os resultados: 5, 3, 4, 2,
>> 6, 1, 1, 3, 1, 4, 2, 3, 5, 6, 3, 4, 5, 4, 4, 4 e neste ponto Zé Roberto se
>> declarou vitorioso. Sábado de tarde o dado teve os resultados: 6, 1, 4, 2,
>> 3, 5, 6, 6; neste momento o jogo foi interrompido pela queda de um
>> meteorito. Quando a situação se acalmou, eles concordaram em continuar do
>> ponto em que estavam. Qual é a probabilidade de que Zé Roberto seja o
>> vencedor?
>>
>> --
>> Esta mensagem foi verificada pelo sistema de antivírus e
>> acredita-se estar livre de perigo.
>
>
> --
> Esta mensagem foi verificada pelo sistema de antivírus e
> acredita-se estar livre de perigo.

-- 
Esta mensagem foi verificada pelo sistema de antiv�rus e
 acredita-se estar livre de perigo.



[obm-l] Desafio de probabilidade

2020-07-25 Por tôpico Professor Vanderlei Nemitz
Bom dia!
O problema a seguir encontra-se em uma prova de desafios da PUC-RJ, muito
boas!!!
Acho que são organizadas pelo professor Nicolau Saldanha.
Encontrei uma resposta bem alta, mais de 90%. Será que está correto?
Muito obrigado!

Zé Roberto e Umberto gostam de jogar par ou ímpar; Zé Roberto sempre pede
par e Umberto sempre pede íımpar. Eles gostam de inventar novas maneiras de
jogar. A última maneira que eles inventaram usa um dado comum, com seis
faces numeradas de 1 a 6. Eles jogam o dado várias vezes até que um número
saia três vezes seguidas; Zé Roberto ganha se este número for par, Umberto
ganha se for ímpar. Sábado de manhã o dado teve os resultados: 5, 3, 4, 2,
6, 1, 1, 3, 1, 4, 2, 3, 5, 6, 3, 4, 5, 4, 4, 4 e neste ponto Zé Roberto se
declarou vitorioso. Sábado de tarde o dado teve os resultados: 6, 1, 4, 2,
3, 5, 6, 6; neste momento o jogo foi interrompido pela queda de um
meteorito. Quando a situação se acalmou, eles concordaram em continuar do
ponto em que estavam. Qual é a probabilidade de que Zé Roberto seja o
vencedor?

-- 
Esta mensagem foi verificada pelo sistema de antiv�rus e
 acredita-se estar livre de perigo.



Re: [obm-l] Problema simples gera um complicado?

2020-07-02 Por tôpico Professor Vanderlei Nemitz
Muito obrigado, Matheus!
Vou estudar sobre esse ponto especial!

Em qui., 2 de jul. de 2020 às 19:58, Matheus Bezerra <
matheusbezerr...@gmail.com> escreveu:

> Olá Vanderlei, boa noite. Esse é um fato conhecido, essas retas concorrem
> em um ponto chamado de Ponto de Fermat. Pesquisa por isso que você deve
> encontrar alguma prova. ;)
>
> *Matheus BL*
>
>
> Em qui., 2 de jul. de 2020 às 18:55, Professor Vanderlei Nemitz <
> vanderma...@gmail.com> escreveu:
>
>> Oi, pessoal, tudo bem?
>>
>> Resolvi um problema simples, que me fez pensar em outro, talvez
>> complicado. Bom, pelos menos são encontrei uma solução. Será que é verdade?
>> Se alguém puder ajudar a provar, caso seja, ficarei muito agradecido. Sem
>> querer "exigir" nada, afinal de contas eu não consegui, mas se puder ser
>> sem usar geometria analítica... :)
>>
>> Sobre os lados de um triângulo ABC são construídos externamente os
>> triângulos equiláteros BCD, CAE e ABF. Prove que os segmentos AD, BE e CF
>> são congruentes. (Esse é o problema simples!)
>>
>> Gostaria de provar que:
>>
>> *Sobre os lados de um triângulo ABC são construídos externamente os
>> triângulos equiláteros BCD, CAE e ABF. Prove que os segmentos AD, BE e CF
>> (ou suas retas suporte) concorrem em um mesmo ponto.*
>>
>> Muito obrigado!
>>
>> --
>> Esta mensagem foi verificada pelo sistema de antivírus e
>> acredita-se estar livre de perigo.
>
>
> --
> Esta mensagem foi verificada pelo sistema de antivírus e
> acredita-se estar livre de perigo.

-- 
Esta mensagem foi verificada pelo sistema de antiv�rus e
 acredita-se estar livre de perigo.



[obm-l] Problema simples gera um complicado?

2020-07-02 Por tôpico Professor Vanderlei Nemitz
Oi, pessoal, tudo bem?

Resolvi um problema simples, que me fez pensar em outro, talvez complicado.
Bom, pelos menos são encontrei uma solução. Será que é verdade? Se alguém
puder ajudar a provar, caso seja, ficarei muito agradecido. Sem querer
"exigir" nada, afinal de contas eu não consegui, mas se puder ser sem usar
geometria analítica... :)

Sobre os lados de um triângulo ABC são construídos externamente os
triângulos equiláteros BCD, CAE e ABF. Prove que os segmentos AD, BE e CF
são congruentes. (Esse é o problema simples!)

Gostaria de provar que:

*Sobre os lados de um triângulo ABC são construídos externamente os
triângulos equiláteros BCD, CAE e ABF. Prove que os segmentos AD, BE e CF
(ou suas retas suporte) concorrem em um mesmo ponto.*

Muito obrigado!

-- 
Esta mensagem foi verificada pelo sistema de antiv�rus e
 acredita-se estar livre de perigo.



[obm-l] Problema de Geometria plana

2020-05-11 Por tôpico Vanderlei Nemitz
Boa noite!
Vi esse problema em uma lista, mas talvez tenha alguma falha no enunciado.
Ou será no leitor?
Muito obrigado!

*Seja ABC um triângulo e D um ponto sobre o lado AC tal que AB = CD. Sejam
E e F os pontos médios de AD e BC, respectivamente. Se a reta BA intersecta
a reta FE em M, prove que AM = ME*.

-- 
Esta mensagem foi verificada pelo sistema de antiv�rus e
 acredita-se estar livre de perigo.



[obm-l] Enunciado confuso de combinatória, ou não?

2020-05-10 Por tôpico Vanderlei Nemitz
Boa tarde!

Não entendi o enunciado da seguinte questão.
A ordem das cores escolhida no cubo "inicial" é fixa?
Faz diferença pintar ou não os vértices?
Ele pensou simplesmente em escolher 3 vértices dos 8?

Muito obrigado!




*Considere um cubo ABCDEFGH no qual ABCD é uma face com 16 cm² de área, AE
e BH são arestas e AG é uma diagonal do cubo.Considere que, em cada um de
seus vértices, serão pintados três triângulos retângulos de mesma cor, cada
um sobre uma das faces para as quais aquele vértice é comum, com o vértice
do ângulo reto sendo o vértice do cubo, e com 0,4 cm em cada um de seus
catetos. Cada um dos vértices será pintado em uma única cor, distinta de
todas as outras. A partir daí, serão escolhidos três de seus vértices para
que se faça uma truncagem do cubo. Truncar um sólido significa fazer nele
um ou mais cortes planos. Neste caso, serão feitos exatamente três cortes
planos sobre arestas que convergem em um mesmo vértice, e tais cortes serão
feitos a 0,4 cm de distância dos vértices escolhidos. Calcule o total de
poliedros distintos que se pode obter, a partir do cubo, ao fazer os cortes
citados, considerando que um poliedro difere de outro também pelas cores
nas quais alguns de seus vértices estãopintados. RESPOSTA = 56*

-- 
Esta mensagem foi verificada pelo sistema de antiv�rus e
 acredita-se estar livre de perigo.



[obm-l] Re: [obm-l] Re: [obm-l] Produtório trigonométrico

2020-05-10 Por tôpico Vanderlei Nemitz
Boa tarde, Anderson!
Depois que postei pensei em sen(3x)/senx, que é equivalente a 1 + 2.cos
(2x).
Daí fica "tranquilo"!
Muito obrigado!



Em sáb., 9 de mai. de 2020 às 18:36, Anderson Torres <
torres.anderson...@gmail.com> escreveu:

>
>
> Em qui., 7 de mai. de 2020 às 15:19, Vanderlei Nemitz <
> vanderma...@gmail.com> escreveu:
>
>> Boa tarde!
>> Alguém tem uma ideia para o seguinte produto?
>> Tentei diversas transformações, mas sem sucesso.
>>
>> A reposta é 1.
>>
>> Produtório para k variando de 1 a n de (1 + 2.cos[(2pi.3^k)/(3^n + 1)]).
>>
>
> Vou tentar fazer aqui em casa, mas um caminho pronmissor é o seguinte:
>
> 1+2 cos(x) = 2 . (1/2 + cos (x)) =  2 . (cos pi/3 + cos (x)) = 2
> cos(pi/6+x/2) cos(pi/6-x/2)
>
> A coisa toda vira um produto gigante, e temos que demonstrar que isso é
> igual a 1.
>
>
>
>>
>> Espero que tenha ficado clara a escrita.
>>
>> Muito obrigado!
>>
>>
>> <https://www.avast.com/sig-email?utm_medium=email_source=link_campaign=sig-email_content=webmail>
>>  Livre
>> de vírus. www.avast.com
>> <https://www.avast.com/sig-email?utm_medium=email_source=link_campaign=sig-email_content=webmail>.
>>
>> <#m_-7442210811298573756_m_-4423768513225571420_DAB4FAD8-2DD7-40BB-A1B8-4E2AA1F9FDF2>
>>
>> --
>> Esta mensagem foi verificada pelo sistema de antivírus e
>> acredita-se estar livre de perigo.
>
>
> --
> Esta mensagem foi verificada pelo sistema de antivírus e
> acredita-se estar livre de perigo.

-- 
Esta mensagem foi verificada pelo sistema de antiv�rus e
 acredita-se estar livre de perigo.



[obm-l] Produtório trigonométrico

2020-05-07 Por tôpico Vanderlei Nemitz
Boa tarde!
Alguém tem uma ideia para o seguinte produto?
Tentei diversas transformações, mas sem sucesso.

A reposta é 1.

Produtório para k variando de 1 a n de (1 + 2.cos[(2pi.3^k)/(3^n + 1)]).

Espero que tenha ficado clara a escrita.

Muito obrigado!


Livre
de vírus. www.avast.com
.
<#DAB4FAD8-2DD7-40BB-A1B8-4E2AA1F9FDF2>

-- 
Esta mensagem foi verificada pelo sistema de antiv�rus e
 acredita-se estar livre de perigo.



Re: [obm-l] Conjuntos

2020-03-28 Por tôpico Vanderlei Nemitz
Matheus, como não pensei nisso?
hehehehe

Muito obrigado, bela solução!

Em sáb., 28 de mar. de 2020 às 10:48, Matheus Henrique <
matheushss2...@gmail.com> escreveu:

> Note que a soma dos elementos do conjunto é igual a 30*31/2=465
> 465-232=233,
> Denotemos por A um subconjunto de {1,2,3...30} e por A' os complemento
> desse subconjunto,isto é,os elementos que não fazem parte de A.
> Chamemos S(A) a soma dos elementos do conjunto A.
> É fácil ver que S(A)+S(A')=435.
> Mas se S(A)>232,logo,S(A')<=232,desse maneira,para conjunto o qual
> S(A)>232,existe um conjunto A',tal que S(A')<=232.
> Conclue-se assim, que o número de elementos que satisfazem o enunciado é
> igual à metade do total de subconjuntos,como existem 2^30 subconjuntos,há
> 2^29 conjuntos que satisfazem o enunciado.
>
> Em sáb., 28 de mar. de 2020 às 07:42, Vanderlei Nemitz <
> vanderma...@gmail.com> escreveu:
>
>> Bom dia, pessoal!
>> Alguém teria uma ideia bacana para esse problema?
>> Muito obrigado!
>>
>> *Quantos subconjuntos do conjunto {1, 2, 3, ..., 30} têm a propriedade de
>> que a soma de seus elementos seja maior do que 232?*
>>
>> --
>> Esta mensagem foi verificada pelo sistema de antivírus e
>> acredita-se estar livre de perigo.
>
>
> --
> Esta mensagem foi verificada pelo sistema de antivírus e
> acredita-se estar livre de perigo.

-- 
Esta mensagem foi verificada pelo sistema de antiv�rus e
 acredita-se estar livre de perigo.



[obm-l] Conjuntos

2020-03-28 Por tôpico Vanderlei Nemitz
Bom dia, pessoal!
Alguém teria uma ideia bacana para esse problema?
Muito obrigado!

*Quantos subconjuntos do conjunto {1, 2, 3, ..., 30} têm a propriedade de
que a soma de seus elementos seja maior do que 232?*

-- 
Esta mensagem foi verificada pelo sistema de antiv�rus e
 acredita-se estar livre de perigo.



[obm-l] Combinatória

2020-03-13 Por tôpico Vanderlei Nemitz
Bom dia!
Não sei se minha mensagem chegou para vocês.
Por via das dúvidas, te encaminho.

Alguém tem uma ideia para esse problema?

Muito obrigado!

De quantos modos se podem sentar em fila, 3 ingleses, 3 franceses e 3
turcos, de modo que não fiquem dois compatriotas juntos?


A resposta é 37584.

-- 
Esta mensagem foi verificada pelo sistema de antiv�rus e
 acredita-se estar livre de perigo.



[obm-l] Combinatória

2020-03-11 Por tôpico Vanderlei Nemitz
Boa noite!
Alguém tem uma ideia para esse problema?

Muito obrigado!

De quantos modos se podem sentar em fila, 3 ingleses, 3 franceses e 3
turcos, de modo que não fiquem dois compatriotas juntos?


A resposta é 37584.

-- 
Esta mensagem foi verificada pelo sistema de antiv�rus e
 acredita-se estar livre de perigo.



[obm-l] Soma de raízes quadradas

2020-02-17 Por tôpico Vanderlei Nemitz
Boa tarde!
Existe uma fórmula fechada para a soma das raízes quadradas dos n primeiros
números naturais?

Muito obrigado!

-- 
Esta mensagem foi verificada pelo sistema de antiv�rus e
 acredita-se estar livre de perigo.



[obm-l] Ângulos em um triângulo

2020-02-13 Por tôpico Vanderlei Nemitz
Boa noite!

Usei várias leis dos senos, obtive coisas legais, mas não o ângulo pedido.
Alguém conhece algo interessante?



Muito obrigado!



*Em um triângulo ABC, os pontos consecutivos M, Q, N do lado AC são tais
que AM = NC. Se Q é ponto médio de MN e os ângulos NBC e ABM medem 20º,
calcule a medida do ângulo BQC.*

-- 
Esta mensagem foi verificada pelo sistema de antiv�rus e
 acredita-se estar livre de perigo.



[obm-l]

2020-02-13 Por tôpico Vanderlei Nemitz
Usei várias leis dos senos, obtive coisas legais, mas não o ângulo pedido.
Alguém conhece algo interessante?

Muito obrigado!

Em um triângulo ABC, em AC localiza-se os pontos consecutivos M,Q e N, tal
que AM=NC. Se Q é ponto médio de MN e os ângulos NBC e ABM medem 20º,
calcule a medida do ângulo BQC.

-- 
Esta mensagem foi verificada pelo sistema de antiv�rus e
 acredita-se estar livre de perigo.



[obm-l] Re: [obm-l] Re: [obm-l] Soma de raízes cúbicas de cossenos

2020-01-24 Por tôpico Vanderlei Nemitz
Como?

Não entendi a ideia...


Em sex, 24 de jan de 2020 02:37, saulo nilson 
escreveu:

> COS 15=COS 30/2
> COS 15=COS(3*5)
> DAÍ ENCONTRA O VALOR DE COS5 =COS10/2
> DAÍ ENCONTRA O VALOR DE COS 10
>
> S= F(COS 10) QUE ENCONTRA O VALOR
>
> On Sun, Jan 19, 2020 at 8:41 AM Vanderlei Nemitz 
> wrote:
>
>> Bom dia, pessoal!
>>
>> Pensei em resolver a seguinte questão associando cos 40°, cos 80° e cos
>> 160° às raízes da equação cos(3x) = -1/2 e utilizando o arco triplo,
>> recaindo em uma equação de grau 3. Porém, fica difícil determinar o produto
>> de 2 em 2 das raízes cúbicas. Alguém conhece uma solução melhor?
>> Muito obrigado!
>>
>> S = (cos 40°)^(1/3) + (cos 80°)^(1/3) +  (cos 160°)^(1/3)
>>
>> (Soma das raízes cúbicas de cos 40°, cos 80° e cos 160°)
>>
>> --
>> Esta mensagem foi verificada pelo sistema de antivírus e
>> acredita-se estar livre de perigo.
>
>
> --
> Esta mensagem foi verificada pelo sistema de antivírus e
> acredita-se estar livre de perigo.

-- 
Esta mensagem foi verificada pelo sistema de antiv�rus e
 acredita-se estar livre de perigo.



[obm-l] Soma de raízes cúbicas de cossenos

2020-01-19 Por tôpico Vanderlei Nemitz
Bom dia, pessoal!

Pensei em resolver a seguinte questão associando cos 40°, cos 80° e cos
160° às raízes da equação cos(3x) = -1/2 e utilizando o arco triplo,
recaindo em uma equação de grau 3. Porém, fica difícil determinar o produto
de 2 em 2 das raízes cúbicas. Alguém conhece uma solução melhor?
Muito obrigado!

S = (cos 40°)^(1/3) + (cos 80°)^(1/3) +  (cos 160°)^(1/3)

(Soma das raízes cúbicas de cos 40°, cos 80° e cos 160°)

-- 
Esta mensagem foi verificada pelo sistema de antiv�rus e
 acredita-se estar livre de perigo.



[obm-l] Re: [obm-l] Re: [obm-l] Potência

2020-01-11 Por tôpico Vanderlei Nemitz
Está em um livro na parte de potenciação.
Mas mesmo assim, como faria com essa ideia?

Em sáb, 11 de jan de 2020 11:18, Esdras Muniz 
escreveu:

> Acho que é d) 04
>
> Em sáb, 11 de jan de 2020 11:01, Esdras Muniz 
> escreveu:
>
>> Pode usar a função fi.
>>
>> Em sáb, 11 de jan de 2020 10:23, Vanderlei Nemitz 
>> escreveu:
>>
>>> Bom dia!
>>> Eu resolvi essa questão, mas creio que trabalhei demais!
>>>
>>> Alguém conhece um modo relativamente simples?
>>>
>>> Os dois últimos algarismos de 2^222 são:
>>> a) 84
>>> b) 24
>>> c) 64
>>> d) 04
>>> e) 44
>>>
>>> Muito obrigado!
>>>
>>> Vanderlei
>>>
>>> --
>>> Esta mensagem foi verificada pelo sistema de antivírus e
>>> acredita-se estar livre de perigo.
>>
>>
> --
> Esta mensagem foi verificada pelo sistema de antivírus e
> acredita-se estar livre de perigo.

-- 
Esta mensagem foi verificada pelo sistema de antiv�rus e
 acredita-se estar livre de perigo.



[obm-l] Potência

2020-01-11 Por tôpico Vanderlei Nemitz
Bom dia!
Eu resolvi essa questão, mas creio que trabalhei demais!

Alguém conhece um modo relativamente simples?

Os dois últimos algarismos de 2^222 são:
a) 84
b) 24
c) 64
d) 04
e) 44

Muito obrigado!

Vanderlei

-- 
Esta mensagem foi verificada pelo sistema de antiv�rus e
 acredita-se estar livre de perigo.



Re: [obm-l] Anagramas

2019-09-25 Por tôpico Vanderlei Nemitz
Se puder, poste sua resolução.

Muito obrigado!

Em qua, 25 de set de 2019 15:02, Mauricio de Araujo <
mauricio.de.ara...@gmail.com> escreveu:

> boa tarde!
> eu pensei em usar o principio da inclusão-enclusão... achei 14766...
>
> Att,
> __
> Mauricio de Araujo
> 
>
>
> Em sáb, 21 de set de 2019 às 21:25, Vanderlei Nemitz <
> vanderma...@gmail.com> escreveu:
>
>> Boa noite pessoal!
>>
>> Resolvi uma questão e obtive como resposta 12246.
>> Gostaria de saber se está correta.
>>
>> Quantos anagramas da palavra CONFIRMAR apresenta as letras de pelo menos
>> uma sílaba da palavra original juntas e em ordem? Por exemplo, C*FIR*AMORN
>> e *MARCONFIR* são anagramas válidos, mas FOCMRAIRN não é.
>>
>> Muito obrigado!
>>
>>
>>
>> --
>> Esta mensagem foi verificada pelo sistema de antivírus e
>> acredita-se estar livre de perigo.
>
>
> --
> Esta mensagem foi verificada pelo sistema de antivírus e
> acredita-se estar livre de perigo.

-- 
Esta mensagem foi verificada pelo sistema de antiv�rus e
 acredita-se estar livre de perigo.



[obm-l] Anagramas

2019-09-21 Por tôpico Vanderlei Nemitz
Boa noite pessoal!

Resolvi uma questão e obtive como resposta 12246.
Gostaria de saber se está correta.

Quantos anagramas da palavra CONFIRMAR apresenta as letras de pelo menos
uma sílaba da palavra original juntas e em ordem? Por exemplo, C*FIR*AMORN
e *MARCONFIR* são anagramas válidos, mas FOCMRAIRN não é.

Muito obrigado!

-- 
Esta mensagem foi verificada pelo sistema de antiv�rus e
 acredita-se estar livre de perigo.



Re: [obm-l] Determinante

2019-07-24 Por tôpico Vanderlei Nemitz
Muito obrigado, Ralph!
Confesso que ontem, 30 minutos depois de postar a pergunta, tive essa ideia
da soma das raízes.
Mesmo assim, acho uma ótima questão para dividir com o grupo.

Um abraço!

Em qua, 24 de jul de 2019 12:26, Ralph Teixeira 
escreveu:

> Ah, tenho uma ideia rapida para a 4a raiz: note que o termo em z^3 nao
> existe... Entao a soma das raizes eh 0. Assim, se z1=w, z2=x e z3=y, entao
> devemos ter z4=-w-x-y.
>
> Abraco, Ralph.
>
> On Wed, Jul 24, 2019 at 11:22 AM Ralph Teixeira  wrote:
>
>> Eu entendi a dica assim: finja momentanemante (apenas para ajudar a
>> pensar) que x, y e w sao constantes, digamos, 3, pi e 111. Entao abrindo o
>> determinante pela ultima coluna, voce vai ficar com um polinomio de quarto
>> grau em z, correto? Pois bem, se as raizes desses polinomio forem z1, z2,
>> z3 e z4, entao o polinomio tem que ser P(z)=a(z-z1)(z-z2)(z-z3)(z-z4), onde
>> a eh o coeficiente de z^4 no polinomio.
>>
>> Entao, vamos fazer isso, pensando que z eh a unica variavel e x,y e w sao
>> constantes. O coeficiente de z^4 eh o determinante 3x3 do canto superior
>> esquerdo, que eh Vandermonde, entao a=(x-w)(y-w)(y-x). Claramente (sim?),
>> z1=w, z2=x e z3=y sao raizes, entao jah temos P(z)=(x-w)(y-w)(y-x).
>> (z-w)(z-x)(z-y). (z-z4). Falta apenas mostrar que z4=-w-x-y eh a ultima
>> raiz, ou seja, se voce mostrar que aquele determinante se anula sempre que
>> x+y+z+w=0, acabou...
>>
>> Abraco, Ralph.
>>
>> On Wed, Jul 24, 2019 at 12:24 AM Vanderlei Nemitz 
>> wrote:
>>
>>> Pessoal, como posso calcular o seguinte determinante, utilizando um
>>> polinômio em z?
>>>
>>> 1   1  1  1
>>> w   x  y   z
>>> w^2   x^2   y^2   z^2
>>> w^4   x^4   y^4   z^4
>>>
>>> A resposta é  (z − y)(z − x)(z − w)(y − x)(y − w)(x − w)(w + x + y + z).
>>>
>>> Vi em uma lista e a dica é essa:
>>> Expanda o determinante ao longo da última coluna e encontre seus zeros
>>> como um polinômio em z.
>>>
>>> Não conheço esse truque.
>>>
>>> Muito obrigado!
>>>
>>>
>>>
>>> --
>>> Esta mensagem foi verificada pelo sistema de antivírus e
>>> acredita-se estar livre de perigo.
>>
>>
> --
> Esta mensagem foi verificada pelo sistema de antivírus e
> acredita-se estar livre de perigo.

-- 
Esta mensagem foi verificada pelo sistema de antiv�rus e
 acredita-se estar livre de perigo.



[obm-l] Determinante

2019-07-23 Por tôpico Vanderlei Nemitz
Pessoal, como posso calcular o seguinte determinante, utilizando um
polinômio em z?

1   1  1  1
w   x  y   z
w^2   x^2   y^2   z^2
w^4   x^4   y^4   z^4

A resposta é  (z − y)(z − x)(z − w)(y − x)(y − w)(x − w)(w + x + y + z).

Vi em uma lista e a dica é essa:
Expanda o determinante ao longo da última coluna e encontre seus zeros como
um polinômio em z.

Não conheço esse truque.

Muito obrigado!

-- 
Esta mensagem foi verificada pelo sistema de antiv�rus e
 acredita-se estar livre de perigo.



[obm-l] Re: [obm-l] Geometria analítica

2019-07-16 Por tôpico Vanderlei Nemitz
Com certeza! É que nesse caso os pontos estão em semiplanos opostos. Talvez
seria isso que eu gostaria de perguntar. Será que nesse caso sim?
Mas e sem derivadas? Será possível resolver? Preciso apresentar a solução
para alunos que não estudaram derivadas...

Muito obrigado!

Em ter, 16 de jul de 2019 às 13:30, Claudio Buffara <
claudio.buff...@gmail.com> escreveu:

> A resposta da 2a questão é NÃO. Pense em M e N próximos um do outro e tão
> distantes da reta que o ângulo MQN é sempre agudo.
>
> Abs
>
> Enviado do meu iPhone
>
> Em 16 de jul de 2019, à(s) 15:44, Vanderlei Nemitz 
> escreveu:
>
> > Pessoal, é possível resolver a seguinte questão sem utilizar
> derivadas?
> >
> > Determinar as coordenadas de um ponto Q pertencente à reta de equação
> y = 3x - 1 tal que a diferença de suas distâncias aos pontos M(4, 1) e
> N(0, 4) seja máxima.
> >
> > A resposta mostra que o triângulo MQN é retângulo em Q, para que a
> diferença seja máxima. Isso ocorre sempre?
> >
> >
> > Muito obrigado!
> >
> > Vander
> >
> > --
> > Esta mensagem foi verificada pelo sistema de antivírus e
> > acredita-se estar livre de perigo.
>
> --
> Esta mensagem foi verificada pelo sistema de antivírus e
>  acredita-se estar livre de perigo.
>
>
> =
> Instru�ões para entrar na lista, sair da lista e usar a lista em
> http://www.mat.puc-rio.br/~obmlistas/obm-l.html
> =
>

-- 
Esta mensagem foi verificada pelo sistema de antiv�rus e
 acredita-se estar livre de perigo.



[obm-l] Geometria analítica

2019-07-16 Por tôpico Vanderlei Nemitz
Pessoal, é possível resolver a seguinte questão sem utilizar derivadas?

Determinar as coordenadas de um ponto Q pertencente à reta de equação y =
3x - 1 tal que a diferença de suas distâncias aos pontos M(4, 1) e N(0, 4)
seja máxima.

A resposta mostra que o triângulo MQN é retângulo em Q, para que a
diferença seja máxima. Isso ocorre sempre?


Muito obrigado!

Vander

-- 
Esta mensagem foi verificada pelo sistema de antiv�rus e
 acredita-se estar livre de perigo.



Re: [obm-l] Bug em probabilidade

2019-06-21 Por tôpico Vanderlei Nemitz
Oi Ralph!
Eu tive o privilégio de assistir na Bienal de Matemática em 2014 um
minicurso seu sobre probabilidades. Desde então minhas ideias mudaram,
creio que para melhor!

Concordo com tudo o que disse e sim, pensei que a segunda maneira é
plausível. Na verdade, essa dúvida surgiu diante da seguinte questão da AFA.

Distribui-se, aleatoriamente, 7 bolas iguais em 3 caixas diferentes.
Sabendo-se que nenhuma delas ficou vazia, a probabilidade de uma caixa
conter, exatamente, 4 bolas é
a) 25%
b) 30%
c) 40%
d) 48%

A resposta proposta é 40%, ou seja, 6/15.

Mas...

Acho que ninguém contestou isso na época.

Seus e-mails foram muito esclarecedores!

Muito obrigado!

Em sex, 21 de jun de 2019 16:44, Ralph Teixeira 
escreveu:

> P.S.: Engracado, eu tambem digitei 3^8... VOCE ME LEVOU PARA O MAU
> CAMINHO! :D :D :D
>
> On Fri, Jun 21, 2019 at 4:36 PM Ralph Teixeira  wrote:
>
>> Oi, Vanderlei.
>>
>> Minha frase predileta, razão de 90% das confusões que fazemos (todos nos,
>> inclusive eu!) em probabilidade:
>>
>> "NEM TODO ESPACO EH EQUIPROVAVEL"
>> ou, traduzindo
>> "SOH PORQUE TEM N MANEIRAS DE ALGO ACONTECER, NAO SIGNIFICA QUE TODAS AS
>> MANEIRAS TEM PROBABILIDADE 1/N".
>>
>> Hm, ok, talvez eu tenha exagerado no CAPS, mas acho que vale a pena. Por
>> conta disso, muita gente (eu tambem!) erra problemas de probabilidade, em
>> varios niveis:
>>
>> a) Obvio: "Eu posso ganhar a mega-sena amanha, ou nao. 50% de chance.
>> oops!"
>> b) Menos obvio: "A soma desses dois dados pode dar de 2 a 12... Entao
>> cada numero tem 1/11 de probabilidade oops!"
>> c) Super sutis: "Tem 2 portas fechadas nestes show, entao cada uma tem
>> 50% de chande de ter um bode oops!"
>>
>> ---///---
>>
>> No seu caso: o que significa "distribuir aleatoriamente" as bolas? Para
>> mim, o mais natural eh pegar cada bola, escolher para ela uma das 3 caixas
>> (com probabilidade 1/3 para cada, que nao estah explicito no enunciado mas
>> parece razoavel)... Entao a sua segunda interpretacao eh a correta, existem
>> 3^8 maneiras EQUIPROVAVEIS de colocar as bolas nas caixas.
>>
>> Note como a outra maneira estah furada: a gente acha mesmo que
>> (x,y,z)=(7,0,0) tem a mesma probabilidade de ocorrer que (x,y,z)=(3,2,2)?
>> Nao, a segunda tripla eh bem mais provavel! Entao nao faz sentido contar
>> numero de solucoes ali.
>>
>> Abraco, Ralph.
>>
>> ---///---
>>
>> P.S.: Eu culpo o jeito que a gente introduz probabilidade no ensino
>> medio O problema eh muitos livros e professores escrevem assim, em
>> letras enormes, cercado por um quadrinho para a gente decorar:
>>
>> * Probabilidade = Numero de Casos Favoraveis / Numero de Casos
>> Possiveis **
>>
>> E isto eh FALSO, porque ele devia escrever antes "SE OS CASOS TEM A MESMA
>> PROBABILIDADE"
>>
>> Abraco, Ralph.
>>
>>
>> On Fri, Jun 21, 2019 at 4:22 PM Vanderlei Nemitz 
>> wrote:
>>
>>> Pessoal, fiquei confuso com a seguinte questão:
>>>
>>> Distribuindo-se aleatoriamente 7 bolas iguais em 3 caixas diferentes,
>>> qual é a probabilidade de que uma delas contenha exatamente 4 bolas?
>>>
>>> Como as bolas são iguais, existem 36 maneiras de alocar as bolas nas
>>> caixas, que é o número de soluções naturais da equação x + y + z = 7.
>>> Dessas, 12 tem 4 bolas em uma das caixas e a probabilidade é 1/3.
>>>
>>> Mas...e se considerarmos que existem 3^8 maneiras de alocar as bolas?
>>> Nesse caso, (C7,4).(2^3).3 = 840 maneiras correspondem a 4 bolas em uma das
>>> caixas e a probabilidade não é 1/3.
>>>
>>> Porque as respostas são diferentes?
>>>
>>> Claro que no primeiro caso as bolas são iguais e no segundo diferentes.
>>>
>>> Mas para uma distribuição aleatória, as probabilidades não deveriam ser
>>> iguais?
>>> Alguém com os olhos fechados, colocando as bolas nas caixas não teria a
>>> mesma chance em qualquer caso?
>>>
>>> Fiquei confuso...
>>>
>>> Errei alguma conta ou não?
>>>
>>> Muito obrigado,
>>>
>>> Vanderlei
>>>
>>> --
>>> Esta mensagem foi verificada pelo sistema de antivírus e
>>> acredita-se estar livre de perigo.
>>
>>
> --
> Esta mensagem foi verificada pelo sistema de antivírus e
> acredita-se estar livre de perigo.

-- 
Esta mensagem foi verificada pelo sistema de antiv�rus e
 acredita-se estar livre de perigo.



[obm-l] Re: Bug em probabilidade

2019-06-21 Por tôpico Vanderlei Nemitz
No e-mail anterior, eu queria dizer 3^7 e não 3^8. Foi um erro de
digitação...

Em sex, 21 de jun de 2019 16:11, Vanderlei Nemitz 
escreveu:

> Pessoal, fiquei confuso com a seguinte questão:
>
> Distribuindo-se aleatoriamente 7 bolas iguais em 3 caixas diferentes, qual
> é a probabilidade de que uma delas contenha exatamente 4 bolas?
>
> Como as bolas são iguais, existem 36 maneiras de alocar as bolas nas
> caixas, que é o número de soluções naturais da equação x + y + z = 7.
> Dessas, 12 tem 4 bolas em uma das caixas e a probabilidade é 1/3.
>
> Mas...e se considerarmos que existem 3^8 maneiras de alocar as bolas?
> Nesse caso, (C7,4).(2^3).3 = 840 maneiras correspondem a 4 bolas em uma das
> caixas e a probabilidade não é 1/3.
>
> Porque as respostas são diferentes?
>
> Claro que no primeiro caso as bolas são iguais e no segundo diferentes.
>
> Mas para uma distribuição aleatória, as probabilidades não deveriam ser
> iguais?
> Alguém com os olhos fechados, colocando as bolas nas caixas não teria a
> mesma chance em qualquer caso?
>
> Fiquei confuso...
>
> Errei alguma conta ou não?
>
> Muito obrigado,
>
> Vanderlei
>

-- 
Esta mensagem foi verificada pelo sistema de antiv�rus e
 acredita-se estar livre de perigo.



[obm-l] Bug em probabilidade

2019-06-21 Por tôpico Vanderlei Nemitz
Pessoal, fiquei confuso com a seguinte questão:

Distribuindo-se aleatoriamente 7 bolas iguais em 3 caixas diferentes, qual
é a probabilidade de que uma delas contenha exatamente 4 bolas?

Como as bolas são iguais, existem 36 maneiras de alocar as bolas nas
caixas, que é o número de soluções naturais da equação x + y + z = 7.
Dessas, 12 tem 4 bolas em uma das caixas e a probabilidade é 1/3.

Mas...e se considerarmos que existem 3^8 maneiras de alocar as bolas? Nesse
caso, (C7,4).(2^3).3 = 840 maneiras correspondem a 4 bolas em uma das
caixas e a probabilidade não é 1/3.

Porque as respostas são diferentes?

Claro que no primeiro caso as bolas são iguais e no segundo diferentes.

Mas para uma distribuição aleatória, as probabilidades não deveriam ser
iguais?
Alguém com os olhos fechados, colocando as bolas nas caixas não teria a
mesma chance em qualquer caso?

Fiquei confuso...

Errei alguma conta ou não?

Muito obrigado,

Vanderlei

-- 
Esta mensagem foi verificada pelo sistema de antiv�rus e
 acredita-se estar livre de perigo.



[obm-l] Re: [obm-l] Re: [obm-l] Combinatória (soma de números)

2019-05-04 Por tôpico Vanderlei Nemitz
Pois é, só penso que o raciocínio não é o mesmo, mas talvez eu esteja
equivocado. Outra coisa, sem querer abusar, já vi em outras questões, mas é
correto chamar os algarismos de 1 a 9 de "significativos" e o 0 não? Não
depende da posição? Com certeza, essa era a intenção do autor,
desconsiderar o zero, creio.
Mas...
*...formados com os n primeiros algarismos significativos...*

<https://www.avast.com/sig-email?utm_medium=email_source=link_campaign=sig-email_content=webmail>
Livre
de vírus. www.avast.com
<https://www.avast.com/sig-email?utm_medium=email_source=link_campaign=sig-email_content=webmail>.
<#DAB4FAD8-2DD7-40BB-A1B8-4E2AA1F9FDF2>

Em sáb, 4 de mai de 2019 às 14:12, Claudio Buffara <
claudio.buff...@gmail.com> escreveu:

> Não vejo porque não. Você vai ter 9!/2 somas iguais a 10.
>
>
> On Sat, May 4, 2019 at 1:51 PM Vanderlei Nemitz 
> wrote:
>
>> Prezados colegas da lista, a seguinte questão é do IME - RJ, do ano de
>> 1957/1958.
>> Gostaria de saber se minha resposta está correta, pois fiquei em dúvida
>> quando forem utilizados os algarismos de 1 a 9, embora a fórmula "funcione".
>>
>> *Determinar a expressão da soma de todos os números de n algarismos,
>> formados com os n primeiros algarismos significativos.  *
>>
>> Inicialmente, pensei que se trata das permutações simples dos algarismos
>> de 1 a n. Por exemplo, se utilizarmos os algarismos de 1 a 5. A soma do
>> menor com o maior números formados é 12345 + 54321 = 6. Como existem 60
>> somas iguais a essa, a soma de todos é 6 x 60 = 360. Analogamente,
>> para n de 1 a 8, temos que a soma é (n + 1)/9 * 99...9 * n!/2 = [(n + 1)! *
>> (10^n - 1)]/18. Para n = 9 isso funciona, mas não segue a mesma lógica,
>> pois 10 não é um algarismo. Alguém poderia confirmar se pensei corretamente
>> e também esse caso isolado?
>>
>> Muito obrigado!
>>
>> Vanderlei
>>
>>
>>
>>
>> <https://www.avast.com/sig-email?utm_medium=email_source=link_campaign=sig-email_content=webmail>
>>  Livre
>> de vírus. www.avast.com
>> <https://www.avast.com/sig-email?utm_medium=email_source=link_campaign=sig-email_content=webmail>.
>>
>> <#m_-6684176662232703957_m_8245286071011598569_DAB4FAD8-2DD7-40BB-A1B8-4E2AA1F9FDF2>
>>
>> --
>> Esta mensagem foi verificada pelo sistema de antivírus e
>> acredita-se estar livre de perigo.
>
>
> --
> Esta mensagem foi verificada pelo sistema de antivírus e
> acredita-se estar livre de perigo.

-- 
Esta mensagem foi verificada pelo sistema de antiv�rus e
 acredita-se estar livre de perigo.



[obm-l] Combinatória (soma de números)

2019-05-04 Por tôpico Vanderlei Nemitz
Prezados colegas da lista, a seguinte questão é do IME - RJ, do ano de
1957/1958.
Gostaria de saber se minha resposta está correta, pois fiquei em dúvida
quando forem utilizados os algarismos de 1 a 9, embora a fórmula "funcione".

*Determinar a expressão da soma de todos os números de n algarismos,
formados com os n primeiros algarismos significativos.  *

Inicialmente, pensei que se trata das permutações simples dos algarismos de
1 a n. Por exemplo, se utilizarmos os algarismos de 1 a 5. A soma do menor
com o maior números formados é 12345 + 54321 = 6. Como existem 60 somas
iguais a essa, a soma de todos é 6 x 60 = 360. Analogamente, para n
de 1 a 8, temos que a soma é (n + 1)/9 * 99...9 * n!/2 = [(n + 1)! * (10^n
- 1)]/18. Para n = 9 isso funciona, mas não segue a mesma lógica, pois 10
não é um algarismo. Alguém poderia confirmar se pensei corretamente e
também esse caso isolado?

Muito obrigado!

Vanderlei




Livre
de vírus. www.avast.com
.
<#DAB4FAD8-2DD7-40BB-A1B8-4E2AA1F9FDF2>

-- 
Esta mensagem foi verificada pelo sistema de antiv�rus e
 acredita-se estar livre de perigo.



[obm-l] Combinatória (permutações)

2019-04-25 Por tôpico Vanderlei Nemitz
Bom dia!

Resolvi a questão a seguir, encontrei como resposta 229080, mas encontrei
essa resposta em uma lista e 133800 em outra. Gostaria de confirmar qual é
a correta. Para mim, 133800 é o número de permutações em que pelo menos um
algarismo par permanece em sua posição original.

Muito obrigado!


*De quantas maneiras podemos permutar os inteiros 1, 2, 3, 4, 5, 6, 7, 8, 9
de forma que nenhum inteiro par fique em sua posição natural?*

-- 
Esta mensagem foi verificada pelo sistema de antiv�rus e
 acredita-se estar livre de perigo.



[obm-l] Re: [obm-l] Re: [obm-l] Polinômio

2019-03-23 Por tôpico Vanderlei Nemitz
Muito obrigado, Claudio e Ralph!
Soluções por demais elegantes!

Eu tinha pensado algo parecido, porém estava tentando encontrar o termo em
x daquele novo polinômio, divido por a de um modo bem mais difícil, como
uma soma de várias PG. Enfim, bem mais trabalhoso e não eficiente.
Um abraço!


Em qua, 20 de mar de 2019 23:58, Ralph Teixeira 
escreveu:

> Um jeito de fazer eh ir direto no polinomio interpolador de Lagrange e
> fazer as contas.
> (https://en.wikipedia.org/wiki/Lagrange_polynomial)
>
> Outro jeito que parece mais elegante (mas no final das contas eh a mesma
> coisa): o polinomio xP(x)-1 tem grau n+1 e todos aqueles n+1 numeros sao
> raizes dele. Entao:
>
> xP(x)-1=a(x-1)(x-2)...(x-2^n)
>
> Tomando x=0 ali em cima, descobre-se que a(-1)(-2)...(-2^n)=-1.
>
> Portanto, P(x)=[a(x-1)...(x-2^n)+1] / x, o que pode parecer estranho, mas
> lembre que o numerador vai ter um "x" para fatorar e cortar com o
> denominador (o a que calculamos garante exatamente isso!). Entao P(0) eh o
> termo indepenente do que sobra depois que cortar o x, isto eh, a soma dos
> coeficientes em x do numerador... bom, para obter um termo em x, voce vai
> ter que multiplicar todos a*(-1)*(-2)*...*(-2^n), EXCETO uma das potencias
> (pois o x entra no lugar dela), isto eh, sao varios termos do tipo
> (-1)/(-2^k).x. Portanto o que voce quer eh
>
> P(0)=SOMA (1/2^k) = 2 - 1/2^n
>
> pois a soma eh uma PG de razao 1/2 com n+1 termos.
>
> Abraco, Ralph.
>
>
>
>
> On Wed, Mar 20, 2019 at 11:08 PM Vanderlei Nemitz 
> wrote:
>
>> Alguém tem uma dica para esse problema?
>> Muito obrigado!
>>
>> *Seja P(x) é um polinômio de grau n tal que P(k) = 1/k para k = 1, 2,
>> 2^2, ..., 2^n. Determine o valor de P(0) em função de n.*
>>
>> --
>> Esta mensagem foi verificada pelo sistema de antivírus e
>> acredita-se estar livre de perigo.
>
>
> --
> Esta mensagem foi verificada pelo sistema de antivírus e
> acredita-se estar livre de perigo.

-- 
Esta mensagem foi verificada pelo sistema de antiv�rus e
 acredita-se estar livre de perigo.



[obm-l] Polinômio

2019-03-20 Por tôpico Vanderlei Nemitz
Alguém tem uma dica para esse problema?
Muito obrigado!

*Seja P(x) é um polinômio de grau n tal que P(k) = 1/k para k = 1, 2, 2^2,
..., 2^n. Determine o valor de P(0) em função de n.*

-- 
Esta mensagem foi verificada pelo sistema de antiv�rus e
 acredita-se estar livre de perigo.



[obm-l] Re: Característica

2019-03-11 Por tôpico Vanderlei Nemitz
Bom dia, pessoal!
Alguém chegou a pensar nessa questão que enviei?
Eu consigo apenas pensar que o posto da matriz dos coeficientes do sistema
é 1.

Muito obrigado!

Em ter, 5 de mar de 2019 às 07:06, Vanderlei Nemitz 
escreveu:

> Na seguinte questão, consigo pensar em um sistema com 2 variáveis livres,
> mas não com apenas 1. De acordo com o gabarito, a resposta é c.
>
> É possível um sistema que satisfaça esse enunciado?
>
>
> Obrigado!
>
>
> *Um sistema linear homogêneo de três equações e três incógnitas admite
> como soluções os ternos (1, 3, 5) e (2, 4, 5), mas não o terno (1, 1, 1). A
> característica do sistema é:*
>
> *a) 0  b) 1  c) 2  d) 3  e) nenhuma das
> respostas anteriores*
>
>
> <https://www.avast.com/sig-email?utm_medium=email_source=link_campaign=sig-email_content=webmail>
>  Livre
> de vírus. www.avast.com
> <https://www.avast.com/sig-email?utm_medium=email_source=link_campaign=sig-email_content=webmail>.
> <#m_7884441411983649374_DAB4FAD8-2DD7-40BB-A1B8-4E2AA1F9FDF2>
>

-- 
Esta mensagem foi verificada pelo sistema de antiv�rus e
 acredita-se estar livre de perigo.



[obm-l] TRIÂNGULO

2019-03-07 Por tôpico Vanderlei Nemitz
Só enxerguei uma saída usando geometria analítica. Alguma ideia?
Muito obrigado!

*Dado um triângulo ABC, com Â= 90º, D é o ponto médio de BC, F é o ponto
médio de AB, E é o ponto médio de AF e G o ponto médio de FB. AD intersecta
CE, CF, CG em P, Q e R respectivamente. Determine a razão PQ/QR.*






Livre
de vírus. www.avast.com
.
<#DAB4FAD8-2DD7-40BB-A1B8-4E2AA1F9FDF2>

-- 
Esta mensagem foi verificada pelo sistema de antiv�rus e
 acredita-se estar livre de perigo.



Re: [obm-l] Escalonamento "estranho"

2019-03-06 Por tôpico Vanderlei Nemitz
Como você escalonaria?
Acredito que eu tenha feito corretamente, mas em algum momento
multiplicamos por algo que depende de k.

Quanto ao nome, não é tão incomum assim! O ITA, por exemplo, chama de
característica.

Muito obrigado!

Em qua, 6 de mar de 2019 12:39, Bernardo Freitas Paulo da Costa <
bernardo...@gmail.com> escreveu:

> On Tue, Mar 5, 2019 at 4:43 PM Vanderlei Nemitz 
> wrote:
> >
> > Boa tarde!
> > Uma questão bem antiga do IME pede para que o sistema linear homogêneo
> seja discutido pelo Teorema de Rouché.
> > (3 - k)x +2y + 2z = 0
> >  x + (4 - k)y +   z = 0
> >2x +4y + (1 + k)z = 0
> >
> > Os valores de k para os quais o determinante da matriz dos coeficientes
> é nulo são k = 1, k = 2 e k = 3, ou seja, para esses valores o sistema é
> indeterminado.
> > Simples!
> >
> > Porém, se resolver pelo teorema de Rouché, para determinar a
> característica da matriz, encontro apenas 2 valores de k para os quais a
> característica é 2. O outro valor "se perde" no caminho.
> > O mesmo acontece se escalonar o sistema.
> >
> > Porque isso ocorre? Pelo fato de multiplicarmos por "zero" em algum
> momento?
>
> Em geral, é muito mais perigoso dividir por zero.  Mas pode ser...
> Como você escalonou esta matriz?
>
>
> Aliás, porque chamar de "caracterísitica" algo cujo nome oficial
> (mesmo em português no Brasil) é chamado de "posto"?  Nunca ouvi isso
> antes, é uma invenção da novlíngua, para confundir e evitar que os
> alunos aprendam lendo a Wikipedia??
>
> Abraços,
> --
> Bernardo Freitas Paulo da Costa
>
> --
> Esta mensagem foi verificada pelo sistema de antivírus e
>  acredita-se estar livre de perigo.
>
>
> =
> Instru�ões para entrar na lista, sair da lista e usar a lista em
> http://www.mat.puc-rio.br/~obmlistas/obm-l.html
> =
>

-- 
Esta mensagem foi verificada pelo sistema de antiv�rus e
 acredita-se estar livre de perigo.



[obm-l] Escalonamento "estranho"

2019-03-05 Por tôpico Vanderlei Nemitz
Boa tarde!
Uma questão bem antiga do IME pede para que o sistema linear homogêneo seja
discutido pelo Teorema de Rouché.
*(3 - k)x +2y + 2z = 0*
* x + (4 - k)y +   z = 0*
*   2x +4y + (1 + k)z = 0*

Os valores de k para os quais o determinante da matriz dos coeficientes é
nulo são k = 1, k = 2 e k = 3, ou seja, para esses valores o sistema é
indeterminado.
Simples!

Porém, se resolver pelo teorema de Rouché, para determinar a característica
da matriz, encontro apenas 2 valores de k para os quais a característica é
2. O outro valor "se perde" no caminho.
O mesmo acontece se escalonar o sistema.

Porque isso ocorre? Pelo fato de multiplicarmos por "zero" em algum momento?
Se sim, como proceder para escalonar o sistema ou a matriz dos coeficientes
para determinar os três valores de k?

Muito obrigado!

Vanderlei

-- 
Esta mensagem foi verificada pelo sistema de antiv�rus e
 acredita-se estar livre de perigo.



[obm-l] Característica

2019-03-05 Por tôpico Vanderlei Nemitz
Na seguinte questão, consigo pensar em um sistema com 2 variáveis livres,
mas não com apenas 1. De acordo com o gabarito, a resposta é c.

É possível um sistema que satisfaça esse enunciado?


Obrigado!


*Um sistema linear homogêneo de três equações e três incógnitas admite como
soluções os ternos (1, 3, 5) e (2, 4, 5), mas não o terno (1, 1, 1). A
característica do sistema é:*

*a) 0  b) 1  c) 2  d) 3  e) nenhuma das
respostas anteriores*


Livre
de vírus. www.avast.com
.
<#DAB4FAD8-2DD7-40BB-A1B8-4E2AA1F9FDF2>

-- 
Esta mensagem foi verificada pelo sistema de antiv�rus e
 acredita-se estar livre de perigo.



Re: [obm-l] Matriz e determinante

2019-02-19 Por tôpico Vanderlei Nemitz
OuMuito obrigado, Cláudio e demais colegas!
Eu pensava que usava "apenas" fatos mais simples. Vi a indicação do artigo
e vou ler, mas queria saber se vocês indicam algum livro ou material para
estudar essas coisas mais "sofisticadas".

Imagino que na seguinte questão também seja necessário usar autovalores.
Muito obrigado!

Considere a matriz
A = 0 1 2
   0 0 1
   0 0 0
Para quantos números naturais n existe uma matriz X tal que X^n = A?
a) 1
b) 2
c) 3
d) infinitos




Em ter, 19 de fev de 2019 15:23, Bernardo Freitas Paulo da Costa <
bernardo...@gmail.com escreveu:

> On Tue, Feb 19, 2019 at 7:45 AM Claudio Buffara
>  wrote:
> >
> > Toda matriz tem um autovalor. De fato, uma matriz nxn tem n autovalores,
> que podem não ser reais e nem todos distintos.
>
> Tem um detalhe que não afeta a validade da sua demonstração, mas acho
> importante ressaltar.  A definição de autovalores que dá n autovalores
> para uma matriz nxn é "algébrica", e não vem necessariamente com n
> autovetores associados (que são ditos "geométricos").  Isso acontece
> com matrizes nilpotentes, por exemplo, e de forma mais geral em blocos
> de Jordan (como o artigo mostra).  O importante (que vem justamente de
> Jordan) é que para cada autovalor k (SEM multiplicidade, portanto
> podendo ser menos do que n) sempre tem pelo menos um autovetor X
> correspondente, e daí você pode usar o seu raciocínio para mostrar que
> Re(k) = 1/2.  Daí você volta para a álgebra, e vê que as
> multiplicidades (algébricas) dos autovalores conjugados são iguais.
> (Talvez até seja possível mostrar que se M é real, as multiplicidades
> geométricas de autovalores conjugados também são iguais, mas neste
> caso não precisa)
>
> > Dá uma olhada nesse artigo aqui:
> https://www.maa.org/sites/default/files/pdf/awards/Axler-Ford-1996.pdf
>
> Muito bom!!  Vou usar nas minhas aulas de Álgebra Linear.  Uma coisa,
> entretanto: não vi a demonstração (nem o enunciado) do fato geométrico
> "há pelo menos um autovetor de verdade para cada autovalor" (e não
> apenas autovetores generalizados).  Não é difícil com tudo o que já
> tem no artigo: por exemplo, (T - lambda * I)^k v = 0 mas não com k-1
> implica que w = (T - lambda I)^{k-1} v é um autovetor "normal" de T.
>
> > []s,
> > Claudio.
> >
> >
> > On Tue, Feb 19, 2019 at 9:23 AM Rodrigo Ângelo 
> wrote:
> >>
> >> Oi, Claudio
> >>
> >> Nesse caso, como a gente sabe que A tem um auto valor k?
> >>
> >> Atenciosamente,
> >> Rodrigo de Castro Ângelo
> >>
> >>
> >> Em seg, 18 de fev de 2019 às 22:25, Claudio Buffara <
> claudio.buff...@gmail.com> escreveu:
> >>>
> >>> Dada uma matriz qualquer M, vou chamar de M* a conjugada transposta de
> M (se M for real, M* = transposta de M).
> >>> Dado um número complexo z, chamarei de z* o conjugado de z.
> >>> E identificarei números complexos com matrizes 1x1.
> >>>
> >>> Seja k um autovalor de A.
> >>> Então existe uma matriz coluna nx1 não nula X tal que AX = kX ==> X*A*
> = k*X*
> >>> X*AX = X*(kX) = kX*X
> >>> X*A*X = (k*X*)X = k*X*X
> >>>
> >>> Somando estas duas equações, obtemos:
> >>> X*AX + X*A*X = (k+k*)X*X ==>
> >>> X*(A + A*)X = 2Re(k)X*X ==>
> >>> X*IX = 2Re(k)X*X ==>
> >>> X*X = 2Re(k)X*X ==>
> >>> (1 - 2Re(k))X*X = 0.
> >>>
> >>> Como X <> 0, X*X > 0 ==> Re(k) = 1/2.
> >>>
> >>> Ou seja, todos os autovalores de A têm parte real = 1/2.
> >>> Como A é real, o polinômio característico de A tem coeficientes reais
> ==>
> >>> os autovalores de A ou são reais (e iguais a 1/2) ou então podem ser
> particionados em pares da forma 1/2 + ib, 1/2 - ib (b real), cujo produto é
> 1/4 + b^2 > 0 ==>
> >>> det(A) = produto dos autovalores de A > 0.
> >>>
> >>> []s,
> >>> Claudio.
> >>>
> >>>
> >>>
> >>>
> >>> On Mon, Feb 18, 2019 at 9:50 PM Vanderlei Nemitz <
> vanderma...@gmail.com> wrote:
> >>>>
> >>>> Pessoal, estou pensando na seguinte questão, consegui alguns
> resultados, mas nada concreto. Alguém com uma ideia que possa resolver?
> >>>>
> >>>> Seja A uma matriz real n x n tal que A + A^t = I.
> >>>> Prove que detA > 0.
> >>>>
> >>>> A^t é a transposta de A.
> >>>>
> >>>> Muito obrigado!
> >>>>
> >>>> Vanderlei
>
>
> --
> Bernardo Freitas Paulo da Costa
>
> --
> Esta mensagem foi verificada pelo sistema de antivírus e
>  acredita-se estar livre de perigo.
>
>
> =
> Instru�ões para entrar na lista, sair da lista e usar a lista em
> http://www.mat.puc-rio.br/~obmlistas/obm-l.html
> =
>

-- 
Esta mensagem foi verificada pelo sistema de antiv�rus e
 acredita-se estar livre de perigo.



[obm-l] Matriz e determinante

2019-02-18 Por tôpico Vanderlei Nemitz
Pessoal, estou pensando na seguinte questão, consegui alguns resultados,
mas nada concreto. Alguém com uma ideia que possa resolver?

*Seja A uma matriz real n x n tal que A + A^t = I.*
*Prove que detA > 0.*

A^t é a transposta de A.

Muito obrigado!

Vanderlei

-- 
Esta mensagem foi verificada pelo sistema de antiv�rus e
 acredita-se estar livre de perigo.



[obm-l] Inequação

2018-11-29 Por tôpico Vanderlei Nemitz
Pessoal, no seguinte problema:

Determine todos os valores do parâmetro real positivo *a* tal que a^cos(2x)
+ a^2.[sen(x)]^2 <= 2 para todo real *x*.
Observação: <= significa "menor do que que ou igual a".

Eu imaginei que para sen(x) = 1, a soma  a^cos(2x) + a^2.[sen(x)]^2, que
pode ser escrita como a^[1 - 2.[sen(x)]^2] + a^2.[sen(x)]^2, é máxima.
Sendo assim, teríamos 1/a + a^2 <= 2, o que implica 0 < a < [-1 +
raiz(5)]/2.

Mas duas coisas:
Está certa essa resposta?
Como mostrar que para sen(x) = 1 a soma é máxima?

Muito obrigado!

-- 
Esta mensagem foi verificada pelo sistema de antiv�rus e
 acredita-se estar livre de perigo.



[obm-l] Re: [obm-l] Re: [obm-l] Re: [obm-l] Demonstração com Geometria Plana?

2018-11-23 Por tôpico Vanderlei Nemitz
Estamos aguardando o Carlos Victor...
:)

Em sex, 23 de nov de 2018 18:14, Mauricio de Araujo <
mauricio.de.ara...@gmail.com escreveu:

> Alguem conseguiu finalizar a demonstração?
>
> Em qua, 21 de nov de 2018 11:52, Vanderlei Nemitz  escreveu:
>
>> Hummm...
>> Parece que prolongando BF e DC, que se encontram num ponto Q, E é o
>> ortocentro do triângulo BDQ.
>> O desenho sugere isso.
>> Mas como mostrar isso?
>>
>> Em ter, 20 de nov de 2018 23:38, Carlos Victor > escreveu:
>>
>>> Oi Vanderlei,
>>>
>>> Uma dica : tente mostrar que o ponto E é o ortocentro de um triângulo "
>>> estratégico". É muito legal que você descubra sozinho
>>>
>>> Abraços
>>>
>>> Carlos Victor
>>>
>>> Em 20/11/2018 17:33, Vanderlei Nemitz escreveu:
>>>
>>> Pessoal, o seguinte problema sai "tranquilamente" usando Geometria
>>> Analítica.
>>> Tentei usar Geometria Plana, mas apenas girei bastante, sem concluir.
>>> Será que é possível?
>>>
>>> Dado um ponto P situado no prolongamento do lado AB de um quadrado ABCD,
>>> traçam-se as retas PC e PD. Pelo ponto E, intersecção de BC e PD,
>>> conduzimos a reta AE cuja intersecção com PC é o ponto F. Provar que BF e
>>> PD são perpendiculares.
>>>
>>> --
>>> Esta mensagem foi verificada pelo sistema de antivrus e
>>> acredita-se estar livre de perigo.
>>>
>>>
>>>
>>
>> --
>> Esta mensagem foi verificada pelo sistema de antivírus e
>> acredita-se estar livre de perigo.
>
>
> --
> Esta mensagem foi verificada pelo sistema de antivírus e
> acredita-se estar livre de perigo.

-- 
Esta mensagem foi verificada pelo sistema de antiv�rus e
 acredita-se estar livre de perigo.



[obm-l] Re: [obm-l] Demonstração com Geometria Plana?

2018-11-21 Por tôpico Vanderlei Nemitz
Hummm...
Parece que prolongando BF e DC, que se encontram num ponto Q, E é o
ortocentro do triângulo BDQ.
O desenho sugere isso.
Mas como mostrar isso?

Em ter, 20 de nov de 2018 23:38, Carlos Victor  Oi Vanderlei,
>
> Uma dica : tente mostrar que o ponto E é o ortocentro de um triângulo "
> estratégico". É muito legal que você descubra sozinho
>
> Abraços
>
> Carlos Victor
>
> Em 20/11/2018 17:33, Vanderlei Nemitz escreveu:
>
> Pessoal, o seguinte problema sai "tranquilamente" usando Geometria
> Analítica.
> Tentei usar Geometria Plana, mas apenas girei bastante, sem concluir. Será
> que é possível?
>
> Dado um ponto P situado no prolongamento do lado AB de um quadrado ABCD,
> traçam-se as retas PC e PD. Pelo ponto E, intersecção de BC e PD,
> conduzimos a reta AE cuja intersecção com PC é o ponto F. Provar que BF e
> PD são perpendiculares.
>
> --
> Esta mensagem foi verificada pelo sistema de antivrus e
> acredita-se estar livre de perigo.
>
>
>

-- 
Esta mensagem foi verificada pelo sistema de antiv�rus e
 acredita-se estar livre de perigo.



[obm-l] Demonstração com Geometria Plana?

2018-11-20 Por tôpico Vanderlei Nemitz
Pessoal, o seguinte problema sai "tranquilamente" usando Geometria
Analítica.
Tentei usar Geometria Plana, mas apenas girei bastante, sem concluir. Será
que é possível?

Dado um ponto P situado no prolongamento do lado AB de um quadrado ABCD,
traçam-se as retas PC e PD. Pelo ponto E, intersecção de BC e PD,
conduzimos a reta AE cuja intersecção com PC é o ponto F. Provar que BF e
PD são perpendiculares.

-- 
Esta mensagem foi verificada pelo sistema de antiv�rus e
 acredita-se estar livre de perigo.



Re: [obm-l] Soma de binomiais

2018-11-20 Por tôpico Vanderlei Nemitz
Muito obrigado, Anderson! Vou estudar o artigo.

Em dom, 18 de nov de 2018 09:50, Anderson Torres <
torres.anderson...@gmail.com escreveu:

> Em qua, 7 de nov de 2018 às 14:38, Vanderlei Nemitz
>  escreveu:
> >
> > Boa tarde!
> > Na seguinte questão, tentei pensar no desenvolvimento de algum binômio,
> em que a parte real fosse a soma S(k), mas não consegui imaginar um.
> Fazendo alguns casos, para k de 1 a 4, conjecturei que S(k) = 2^(2k -
> 1).[2^(2k - 1) + (-1)^k].
> > Mas como posso provar que é verdadeira (se realmente for), a partir do
> zero, de preferência sem usar indução?
> >
> > Outra coisa, depois de obtida a fórmula, como obter o menor k que
> satisfaz o problema sem muitas contas? Eu testei até k igual a 14, usando
> uma calculadora.
> >
> > Obrigado!
> >
> > Seja S(k) = C(4k, 0) + C(4k, 4) + C(4k, 8) + ... + C(4k, 4k), para k =
> 1, 2, 3, 
> > O menor valor de k tal que S(k) é múltiplo de 81, é:
> > a) 7
> > b) 9
> > c) 10
> > d) 12
> > e) 14
> >
>
> Cê pode aprender sobre o Método de Multi-Secção no Problema 4 do
> artigo "Raízes da Unidade", na Eureka! 33:
>
> https://www.obm.org.br/content/uploads/2017/01/Eureka33.pdf
>
> > --
> > Esta mensagem foi verificada pelo sistema de antivírus e
> > acredita-se estar livre de perigo.
>
> --
> Esta mensagem foi verificada pelo sistema de antivírus e
>  acredita-se estar livre de perigo.
>
>
> =
> Instru�ões para entrar na lista, sair da lista e usar a lista em
> http://www.mat.puc-rio.br/~obmlistas/obm-l.html
> =
>

-- 
Esta mensagem foi verificada pelo sistema de antiv�rus e
 acredita-se estar livre de perigo.



Re: [obm-l] Determinante

2018-11-14 Por tôpico Vanderlei Nemitz
Eu iria perguntar a mesma coisa ao Ralph, mas antes eu iria tentar calcular
o determinante mais fácil que ele deixou...

Em qua, 14 de nov de 2018 16:28, Claudio Buffara  Bela sacada!
> Como você pensou nisso?
> O fato da resposta ser (a1)^2 foi uma pista?
>
> Pergunto porque tenho muito interesse por heurística e pela questão "de
> onde vem as idéias matemáticas?"
>
> []s,
> Claudio.
>
>
> On Tue, Nov 13, 2018 at 10:32 PM Ralph Teixeira  wrote:
>
>> Hmm... Que tal olhar para:
>>
>> 0   1   1   1  ...  1
>> 1 z1   0   0  ...  0
>> 1   0  z2  0  ...  0
>> ...
>> 1   0   0   0 ... zn
>>
>> Digo isso porque, elevando esta matriz ao quadrado...
>>
>> Abraco, Ralph.
>>
>> On Tue, Nov 13, 2018 at 3:45 PM Vanderlei Nemitz 
>> wrote:
>>
>>> Agradeço pelas tentativas. Também estou me quebrando nele, mas não
>>> consigo um padrão, apesar de ser fácil concluir o padrão com os resultados
>>> para n igual a 2 e n igual a 3.
>>>
>>>
>>> Em ter, 13 de nov de 2018 15:06, Anderson Torres <
>>> torres.anderson...@gmail.com escreveu:
>>>
>>>>
>>>>
>>>> Em seg, 12 de nov de 2018 às 22:13, Vanderlei Nemitz <
>>>> vanderma...@gmail.com> escreveu:
>>>>
>>>>> Mas será que não é possível provar genericamente?
>>>>>
>>>>
>>>> Eu tentei verificar na internet, mas não achei nada.
>>>> Deve ter algum truquinho que não estou vendo. Talvez uma diagonalizaçao
>>>> esperta...
>>>>
>>>>
>>>>
>>>>>
>>>>> Em seg, 12 de nov de 2018 21:34, Claudio Buffara <
>>>>> claudio.buff...@gmail.com escreveu:
>>>>>
>>>>>> Pruma múltipla escolha, você fez o necessário: testou casos
>>>>>> particulares e eliminou 4 alternativas.
>>>>>>
>>>>>>
>>>>>>
>>>>>> On Mon, Nov 12, 2018 at 7:57 PM Vanderlei Nemitz <
>>>>>> vanderma...@gmail.com> wrote:
>>>>>>
>>>>>>> Gostaria de uma dica na seguinte questão.
>>>>>>> Já tentei muito coisa!
>>>>>>> Desculpe as limitações para digitar o enunciado. Qualquer dúvida,
>>>>>>> estou à disposição.
>>>>>>> Muito obrigado!
>>>>>>>
>>>>>>> Sejam z1, z2, ..., zn as raízes do polinômio complexo P(z) = z^n +
>>>>>>> a(n-1).z^(n - 1) + ... + a1.z + a0, com a0 diferente de 0. Determine o
>>>>>>> valor do determinante da matriz
>>>>>>> n   z1   z2   ... zn
>>>>>>> z1  1 + z1^2 1... 1
>>>>>>> z2  1 1 + z2^2... 1
>>>>>>> 
>>>>>>> zn  1 1 1 + zn^2
>>>>>>>
>>>>>>> a) [a(n-1)]^2
>>>>>>> b) n
>>>>>>> c) 1 + a(n-1) + ... + a1 + a0
>>>>>>> d) (a1)^2
>>>>>>> e) a0
>>>>>>>
>>>>>>> Testei para um polinômio do segundo e outro do terceiro grau e
>>>>>>> obtive a alternativa d, ou seja, o coeficiente a1 elevado ao quadrado.
>>>>>>> Mas como provar?
>>>>>>>
>>>>>>> Muito obrigado!
>>>>>>>
>>>>>>>
>>>>>>> <https://www.avast.com/sig-email?utm_medium=email_source=link_campaign=sig-email_content=webmail>
>>>>>>>  Livre
>>>>>>> de vírus. www.avast.com
>>>>>>> <https://www.avast.com/sig-email?utm_medium=email_source=link_campaign=sig-email_content=webmail>.
>>>>>>>
>>>>>>> <#m_-3496770669724955617_m_1438946694965666227_m_2240385784410830415_m_-2565710904076108649_m_6162287954846621097_m_-4046382275174238934_m_-274481415220420387_m_-3160108196652599415_DAB4FAD8-2DD7-40BB-A1B8-4E2AA1F9FDF2>
>>>>>>>
>>>>>>> --
>>>>>>> Esta mensagem foi verificada pelo sistema de antivírus e
>>>>>>> acredita-se estar livre de perigo.
>>>>>>
>>>>>>
>>>>>> --
>>>>>> Esta mensagem foi verificada pelo sistema de antivírus e
>>>>>> acredita-se estar livre de perigo.
>>>>>
>>>>>
>>>>> --
>>>>> Esta mensagem foi verificada pelo sistema de antivírus e
>>>>> acredita-se estar livre de perigo.
>>>>
>>>>
>>>> --
>>>> Esta mensagem foi verificada pelo sistema de antivírus e
>>>> acredita-se estar livre de perigo.
>>>
>>>
>>> --
>>> Esta mensagem foi verificada pelo sistema de antivírus e
>>> acredita-se estar livre de perigo.
>>
>>
>> --
>> Esta mensagem foi verificada pelo sistema de antivírus e
>> acredita-se estar livre de perigo.
>
>
> --
> Esta mensagem foi verificada pelo sistema de antivírus e
> acredita-se estar livre de perigo.

-- 
Esta mensagem foi verificada pelo sistema de antiv�rus e
 acredita-se estar livre de perigo.



Re: [obm-l] Determinante

2018-11-13 Por tôpico Vanderlei Nemitz
Agradeço pelas tentativas. Também estou me quebrando nele, mas não consigo
um padrão, apesar de ser fácil concluir o padrão com os resultados para n
igual a 2 e n igual a 3.


Em ter, 13 de nov de 2018 15:06, Anderson Torres <
torres.anderson...@gmail.com escreveu:

>
>
> Em seg, 12 de nov de 2018 às 22:13, Vanderlei Nemitz <
> vanderma...@gmail.com> escreveu:
>
>> Mas será que não é possível provar genericamente?
>>
>
> Eu tentei verificar na internet, mas não achei nada.
> Deve ter algum truquinho que não estou vendo. Talvez uma diagonalizaçao
> esperta...
>
>
>
>>
>> Em seg, 12 de nov de 2018 21:34, Claudio Buffara <
>> claudio.buff...@gmail.com escreveu:
>>
>>> Pruma múltipla escolha, você fez o necessário: testou casos particulares
>>> e eliminou 4 alternativas.
>>>
>>>
>>>
>>> On Mon, Nov 12, 2018 at 7:57 PM Vanderlei Nemitz 
>>> wrote:
>>>
>>>> Gostaria de uma dica na seguinte questão.
>>>> Já tentei muito coisa!
>>>> Desculpe as limitações para digitar o enunciado. Qualquer dúvida, estou
>>>> à disposição.
>>>> Muito obrigado!
>>>>
>>>> Sejam z1, z2, ..., zn as raízes do polinômio complexo P(z) = z^n +
>>>> a(n-1).z^(n - 1) + ... + a1.z + a0, com a0 diferente de 0. Determine o
>>>> valor do determinante da matriz
>>>> n   z1   z2   ... zn
>>>> z1  1 + z1^2 1... 1
>>>> z2  1 1 + z2^2... 1
>>>> 
>>>> zn  1 1 1 + zn^2
>>>>
>>>> a) [a(n-1)]^2
>>>> b) n
>>>> c) 1 + a(n-1) + ... + a1 + a0
>>>> d) (a1)^2
>>>> e) a0
>>>>
>>>> Testei para um polinômio do segundo e outro do terceiro grau e obtive a
>>>> alternativa d, ou seja, o coeficiente a1 elevado ao quadrado.
>>>> Mas como provar?
>>>>
>>>> Muito obrigado!
>>>>
>>>>
>>>> <https://www.avast.com/sig-email?utm_medium=email_source=link_campaign=sig-email_content=webmail>
>>>>  Livre
>>>> de vírus. www.avast.com
>>>> <https://www.avast.com/sig-email?utm_medium=email_source=link_campaign=sig-email_content=webmail>.
>>>>
>>>> <#m_-2565710904076108649_m_6162287954846621097_m_-4046382275174238934_m_-274481415220420387_m_-3160108196652599415_DAB4FAD8-2DD7-40BB-A1B8-4E2AA1F9FDF2>
>>>>
>>>> --
>>>> Esta mensagem foi verificada pelo sistema de antivírus e
>>>> acredita-se estar livre de perigo.
>>>
>>>
>>> --
>>> Esta mensagem foi verificada pelo sistema de antivírus e
>>> acredita-se estar livre de perigo.
>>
>>
>> --
>> Esta mensagem foi verificada pelo sistema de antivírus e
>> acredita-se estar livre de perigo.
>
>
> --
> Esta mensagem foi verificada pelo sistema de antivírus e
> acredita-se estar livre de perigo.

-- 
Esta mensagem foi verificada pelo sistema de antiv�rus e
 acredita-se estar livre de perigo.



[obm-l] Re: [obm-l] Re: [obm-l] Transformação

2018-11-12 Por tôpico Vanderlei Nemitz
Boa noite, Claudio!
Muito obrigado pela solução!

Mas fiquei com uma dúvida.
Os resultados de A^(-1) e de A^t não são multiplicações invertidas? Eu
também cheguei nisso, mas pensei que eram coisas diferentes.
A^(-1) = (I - S)*(I + S)^(-1)
A^t = = (I + S)^(-1) * (I - S)

Muito obrigado!


Em sex, 9 de nov de 2018 09:57, Claudio Buffara  Chame a transposta de S de S^t.
> S anti-simétrica ==> S^t = -S
>
> A ortogonal ==> A^t = A^(-1) <==> A*A^t = I
>
> A = (I + S)*(I - S)^(-1) ==>
> A^(-1) = (I - S)*(I + S)^(-1)   (inversa da inversa = matriz original;
> inversa do produto = produto das inversas na ordem oposta)
>
> A^t = ((I - S)^(-1))^t * (I + S)^t   (transposta do produto = produto das
> transpostas na ordem inversa)
> = ((I - S)^(t))^(-1) * (I + S^t)  (transposição e inversão se comutam)
> = (I - S^t)^(-1) * (I + S^t)(transposta da soma = soma das transpostas)
> = (I + S)^(-1) * (I - S)   (S é anti-simétrica)
> = A^(-1)
>
> Logo, A é ortogonal
>
> []s,
> Claudio.
>
>
> On Thu, Nov 8, 2018 at 7:23 PM Vanderlei Nemitz 
> wrote:
>
>> Mostre que se S é uma matriz antissimétrica e A = (I + S).(I - S)^-1, com
>> (I - S) não singular, então A é ortogonal.
>>
>> É possível provar usando conceitos elementares de matrizes?
>>
>> Muito obrigado!
>>
>> (I - S)^-1 é a inversa de I - S.
>>
>> --
>> Esta mensagem foi verificada pelo sistema de antivírus e
>> acredita-se estar livre de perigo.
>
>
> --
> Esta mensagem foi verificada pelo sistema de antivírus e
> acredita-se estar livre de perigo.

-- 
Esta mensagem foi verificada pelo sistema de antiv�rus e
 acredita-se estar livre de perigo.



Re: [obm-l] Determinante

2018-11-12 Por tôpico Vanderlei Nemitz
Mas será que não é possível provar genericamente?

Em seg, 12 de nov de 2018 21:34, Claudio Buffara  Pruma múltipla escolha, você fez o necessário: testou casos particulares e
> eliminou 4 alternativas.
>
>
>
> On Mon, Nov 12, 2018 at 7:57 PM Vanderlei Nemitz 
> wrote:
>
>> Gostaria de uma dica na seguinte questão.
>> Já tentei muito coisa!
>> Desculpe as limitações para digitar o enunciado. Qualquer dúvida, estou à
>> disposição.
>> Muito obrigado!
>>
>> Sejam z1, z2, ..., zn as raízes do polinômio complexo P(z) = z^n +
>> a(n-1).z^(n - 1) + ... + a1.z + a0, com a0 diferente de 0. Determine o
>> valor do determinante da matriz
>> n   z1   z2   ... zn
>> z1  1 + z1^2 1... 1
>> z2  1 1 + z2^2... 1
>> 
>> zn  1 1 1 + zn^2
>>
>> a) [a(n-1)]^2
>> b) n
>> c) 1 + a(n-1) + ... + a1 + a0
>> d) (a1)^2
>> e) a0
>>
>> Testei para um polinômio do segundo e outro do terceiro grau e obtive a
>> alternativa d, ou seja, o coeficiente a1 elevado ao quadrado.
>> Mas como provar?
>>
>> Muito obrigado!
>>
>>
>> <https://www.avast.com/sig-email?utm_medium=email_source=link_campaign=sig-email_content=webmail>
>>  Livre
>> de vírus. www.avast.com
>> <https://www.avast.com/sig-email?utm_medium=email_source=link_campaign=sig-email_content=webmail>.
>>
>> <#m_-274481415220420387_m_-3160108196652599415_DAB4FAD8-2DD7-40BB-A1B8-4E2AA1F9FDF2>
>>
>> --
>> Esta mensagem foi verificada pelo sistema de antivírus e
>> acredita-se estar livre de perigo.
>
>
> --
> Esta mensagem foi verificada pelo sistema de antivírus e
> acredita-se estar livre de perigo.

-- 
Esta mensagem foi verificada pelo sistema de antiv�rus e
 acredita-se estar livre de perigo.



[obm-l] Determinante

2018-11-12 Por tôpico Vanderlei Nemitz
Gostaria de uma dica na seguinte questão.
Já tentei muito coisa!
Desculpe as limitações para digitar o enunciado. Qualquer dúvida, estou à
disposição.
Muito obrigado!

Sejam z1, z2, ..., zn as raízes do polinômio complexo P(z) = z^n +
a(n-1).z^(n - 1) + ... + a1.z + a0, com a0 diferente de 0. Determine o
valor do determinante da matriz
n   z1   z2   ... zn
z1  1 + z1^2 1... 1
z2  1 1 + z2^2... 1

zn  1 1 1 + zn^2

a) [a(n-1)]^2
b) n
c) 1 + a(n-1) + ... + a1 + a0
d) (a1)^2
e) a0

Testei para um polinômio do segundo e outro do terceiro grau e obtive a
alternativa d, ou seja, o coeficiente a1 elevado ao quadrado.
Mas como provar?

Muito obrigado!


Livre
de vírus. www.avast.com
.
<#DAB4FAD8-2DD7-40BB-A1B8-4E2AA1F9FDF2>

-- 
Esta mensagem foi verificada pelo sistema de antiv�rus e
 acredita-se estar livre de perigo.



[obm-l] Transformação

2018-11-08 Por tôpico Vanderlei Nemitz
Mostre que se S é uma matriz antissimétrica e A = (I + S).(I - S)^-1, com
(I - S) não singular, então A é ortogonal.

É possível provar usando conceitos elementares de matrizes?

Muito obrigado!

(I - S)^-1 é a inversa de I - S.

-- 
Esta mensagem foi verificada pelo sistema de antiv�rus e
 acredita-se estar livre de perigo.



[obm-l] Soma de binomiais

2018-11-07 Por tôpico Vanderlei Nemitz
Boa tarde!
Na seguinte questão, tentei pensar no desenvolvimento de algum binômio, em
que a parte real fosse a soma S(k), mas não consegui imaginar um. Fazendo
alguns casos, para k de 1 a 4, conjecturei que S(k) = 2^(2k - 1).[2^(2k -
1) + (-1)^k].
Mas como posso provar que é verdadeira (se realmente for), a partir do
zero, de preferência sem usar indução?

Outra coisa, depois de obtida a fórmula, como obter o menor k que satisfaz
o problema sem muitas contas? Eu testei até k igual a 14, usando uma
calculadora.

Obrigado!

Seja S(k) = C(4k, 0) + C(4k, 4) + C(4k, 8) + ... + C(4k, 4k), para k = 1,
2, 3, 
O menor valor de k tal que S(k) é múltiplo de 81, é:
a) 7
b) 9
c) 10
d) 12
e) 14

-- 
Esta mensagem foi verificada pelo sistema de antiv�rus e
 acredita-se estar livre de perigo.



[obm-l] Re: [obm-l] Re: [obm-l] Questão de probabilidade

2018-11-06 Por tôpico Vanderlei Nemitz
Bela solução, Bruno!
Muito obrigado!

Em ter, 6 de nov de 2018 15:38, Bruno Visnadi  Seja Pa a probabilidade de ocorrência de a. Defina Pb e Pc analogamente.
> a = Pa(1-Pb)(1-Pc)
> b = Pb(1-Pa)(1-Pc)
> c = Pc(1-Pa)(1-Pb)
> p = (1-Pa)(1-Pb)(1-Pc)
> Queremos achar a razão Pa/Pc
> Da equação (a - 2b)p = ab, obtemos:
> (1-Pa)(1-Pb)(1-Pc)²(Pa(1-Pb) - 2Pb(1-Pa)) = PaPb(1-Pa)(1-Pb)(1-Pc)²
> Pa(1-Pb) - 2Pb(1-Pa)  = PaPb
> Pa - 2Pb + PaPb = PaPb
> Pa = 2Pb -> Pb = Pa/2
> Da equação  (b - 3c)p = 2bc, obtemos:
> (1-Pa)²(1-Pb)(1-Pc)(Pb(1-Pc) - 3Pc(1-Pb)) = 2PbPc(1-Pa)²(1-Pb)(1-Pc)
> Pb(1-Pc) - 3Pc(1-Pb) = 2PbPc
> Pb - 3Pc + 2PcPb = 2PbPc
> Pb = 3Pc
> Logo: Pa/2 = 3Pc
> Pa/Pc = 6
>
>
>
>
>
> Em ter, 6 de nov de 2018 às 12:43, Vanderlei Nemitz 
> escreveu:
>
>> Pessoal, alguém tem um ideia de como resolver a seguinte questão? Já
>> tentei muita coisa, sem sucesso.
>> Muito obrigado!
>>
>> Vanderlei
>>
>> Sejam três eventos independentes A, B e C. A probabilidade de que ocorra
>> apenas o evento A é a, apenas o evento B é b e apenas o evento C é c. Seja
>> p a probabilidade de que nenhum dos eventos A, B ou C ocorra. Sabendo que
>> todas as probabilidades citadas são números no intervalo ]0, 1[ e que p
>> satisfaz as equações (a - 2b).p = ab e (b - 3c).p = 2bc, a razão entre a
>> probabilidade de ocorrência de A e a probabilidade de ocorrência de C é:
>> a) 12
>> b) 3
>> c) 10
>> d) 5
>> e) 6
>>
>> --
>> Esta mensagem foi verificada pelo sistema de antivírus e
>> acredita-se estar livre de perigo.
>
>
> --
> Esta mensagem foi verificada pelo sistema de antivírus e
> acredita-se estar livre de perigo.

-- 
Esta mensagem foi verificada pelo sistema de antiv�rus e
 acredita-se estar livre de perigo.



[obm-l] Questão de probabilidade

2018-11-06 Por tôpico Vanderlei Nemitz
Pessoal, alguém tem um ideia de como resolver a seguinte questão? Já tentei
muita coisa, sem sucesso.
Muito obrigado!

Vanderlei

Sejam três eventos independentes A, B e C. A probabilidade de que ocorra
apenas o evento A é a, apenas o evento B é b e apenas o evento C é c. Seja
p a probabilidade de que nenhum dos eventos A, B ou C ocorra. Sabendo que
todas as probabilidades citadas são números no intervalo ]0, 1[ e que p
satisfaz as equações (a - 2b).p = ab e (b - 3c).p = 2bc, a razão entre a
probabilidade de ocorrência de A e a probabilidade de ocorrência de C é:
a) 12
b) 3
c) 10
d) 5
e) 6

-- 
Esta mensagem foi verificada pelo sistema de antiv�rus e
 acredita-se estar livre de perigo.



[obm-l] Re: [obm-l] Re: [obm-l] Re: [obm-l] Dúvida conceitual (equações)

2018-10-15 Por tôpico Vanderlei Nemitz
Claudio:
Eu ficaria com a mesma dúvida!
Pensaria em apenas uma raiz.

Qual é a soma das raízes da equação (cos x)^2 - cos x + 1/4 = 0 no
intervalo [0, 2pi]?

Em seg, 15 de out de 2018 07:00, Claudio Buffara 
escreveu:

> Qual a soma das raizes de (2^x - 8)^3 = 0?
> Se a equação acima fosse apresentada como:
> 2^(3x) - 24*2^(2x) + 192*2^x - 512 = 0,
> isso mudaria sua resposta?
>
> Enviado do meu iPhone
>
> Em 15 de out de 2018, à(s) 00:29, Vanderlei Nemitz 
> escreveu:
>
> Valeu, Pedro! Tomara que mais alguém emita sua opinião.
> Um abraço!
>
> Em dom, 14 de out de 2018 18:59, Pedro José 
> escreveu:
>
>> Boa noite!
>> Bom questionamento. Vou me posicionar na arquibancada.Â
>> Minha posição é controversa. Se quer se levar em conta a repetição
>> tem que se falar do produto das raízes, cada elevada a sua multiplicidade.
>> No caso de soma, cada raiz multiplicada pela multiplicidade.
>> Para esse exemplo, o conjunto solução é {1/2,-1} então o produto é
>> -1/2.
>> Em suma, não aceito n raízes iguais, mas sim uma raiz de multiplicidade
>> n.
>> Se quando queremos provar que algo é unico supomos a existência de dois
>> e provamos que são iguais. Creio que seja contraditório dois ou nais
>> iguais.
>> Mas vamos observar as diversas posições, pois, creio que o assunto não
>> seja pacífico.Â
>> Saudações,Â
>> PJMSÂ
>>
>> Em Dom, 14 de out de 2018 06:33, Vanderlei Nemitz 
>> escreveu:
>>
>>> Bom dia!
>>> Na seguinte questão, que me foi apresentada por um aluno, a resposta
>>> proposta é a alternativa C (1/2). Eu sempre pensei que apenas
>>> considerávamos multiplicidades em equações polinomiais. Como essa é uma
>>> equação exponencial, obtive a resposta B (-1/2). O que é correto pensar?
>>>
>>> O produto das raízes da equação 16.4^3x - 40.4^2x + 17.4^x - 2 = 0 é
>>> igual a:
>>> A) 1
>>> B) - 0,5
>>> C) 0,5
>>> D) - 1
>>> E) 0
>>>
>>> Muito obrigado!
>>>
>>> --
>>> Esta mensagem foi verificada pelo sistema de antivírus e
>>> acredita-se estar livre de perigo.
>>
>>
>> --
>> Esta mensagem foi verificada pelo sistema de antivírus e
>> acredita-se estar livre de perigo.
>
>
> --
> Esta mensagem foi verificada pelo sistema de antivírus e
> acredita-se estar livre de perigo.
>
>
> --
> Esta mensagem foi verificada pelo sistema de antivírus e
> acredita-se estar livre de perigo.
>

-- 
Esta mensagem foi verificada pelo sistema de antiv�rus e
 acredita-se estar livre de perigo.



[obm-l] Re: [obm-l] Re: [obm-l] Dúvida conceitual (equações)

2018-10-14 Por tôpico Vanderlei Nemitz
Valeu, Pedro! Tomara que mais alguém emita sua opinião.
Um abraço!

Em dom, 14 de out de 2018 18:59, Pedro José  escreveu:

> Boa noite!
> Bom questionamento. Vou me posicionar na arquibancada.
> Minha posição é controversa. Se quer se levar em conta a repetição tem que
> se falar do produto das raízes, cada elevada a sua multiplicidade. No caso
> de soma, cada raiz multiplicada pela multiplicidade.
> Para esse exemplo, o conjunto solução é {1/2,-1} então o produto é -1/2.
> Em suma, não aceito n raízes iguais, mas sim uma raiz de multiplicidade n.
> Se quando queremos provar que algo é unico supomos a existência de dois e
> provamos que são iguais. Creio que seja contraditório dois ou nais iguais.
> Mas vamos observar as diversas posições, pois, creio que o assunto não
> seja pacífico.
> Saudações,
> PJMS
>
> Em Dom, 14 de out de 2018 06:33, Vanderlei Nemitz 
> escreveu:
>
>> Bom dia!
>> Na seguinte questão, que me foi apresentada por um aluno, a resposta
>> proposta é a alternativa C (1/2). Eu sempre pensei que apenas
>> considerávamos multiplicidades em equações polinomiais. Como essa é uma
>> equação exponencial, obtive a resposta B (-1/2). O que é correto pensar?
>>
>> O produto das raízes da equação 16.4^3x - 40.4^2x + 17.4^x - 2 = 0 é
>> igual a:
>> A) 1
>> B) - 0,5
>> C) 0,5
>> D) - 1
>> E) 0
>>
>> Muito obrigado!
>>
>> --
>> Esta mensagem foi verificada pelo sistema de antivírus e
>> acredita-se estar livre de perigo.
>
>
> --
> Esta mensagem foi verificada pelo sistema de antivírus e
> acredita-se estar livre de perigo.

-- 
Esta mensagem foi verificada pelo sistema de antiv�rus e
 acredita-se estar livre de perigo.



[obm-l] Dúvida conceitual (equações)

2018-10-14 Por tôpico Vanderlei Nemitz
Bom dia!
Na seguinte questão, que me foi apresentada por um aluno, a resposta
proposta é a alternativa C (1/2). Eu sempre pensei que apenas
considerávamos multiplicidades em equações polinomiais. Como essa é uma
equação exponencial, obtive a resposta B (-1/2). O que é correto pensar?

O produto das raízes da equação 16.4^3x - 40.4^2x + 17.4^x - 2 = 0 é igual
a:
A) 1
B) - 0,5
C) 0,5
D) - 1
E) 0

Muito obrigado!

-- 
Esta mensagem foi verificada pelo sistema de antiv�rus e
 acredita-se estar livre de perigo.



[obm-l] Re: [obm-l] Re: [obm-l] Questão do ITA

2018-10-11 Por tôpico Vanderlei Nemitz
Valeu, Ralph!
Como sempre, uma explicação clara e simples!

Em qua, 10 de out de 2018 17:05, Ralph Teixeira 
escreveu:

> Note que x=5 é um possível valor que resolve aquela equação (mas,
> sinceramente, não interessa, eu faria o raciocínio abaixo com qualquer
> número).
>
> Então qualquer polinômio que satisfaça f(1)=5, f(-1)=10 e f(0)=20
> automaticamente satisfaz todas as condições do enunciado (note que
> a_0=f(0)). Em outras palavras, qualquer polinômio cujo gráfico passe pelos
> pontos (-1,10),(0,20),(1,5) serve.
>
> Agora escolha um ponto (z,0) qualquer como 4o ponto (onde z não é -1, 0
> nem 1). Como quaisquer 4 pontos (com "x"s diferentes) determinam um único
> polinômio de grau 3, haverá um polinômio de grau 3 que passa pelos pontos
> dados e que tem raiz z. Como z pode ser negativo, positivo, raiz(2), ou 42,
> nenhuma das respostas (A)-(D) pode valer (respectivamente!). Então tem que
> ser (E).
>
> Abraço, Ralph.
>
> On Wed, Oct 10, 2018 at 5:41 AM Vanderlei Nemitz 
> wrote:
>
>> Bom dia, pessoal!
>> Encontrei essa questão, que diz ser do ITA (eu particularmente não
>> encontrei na internet).
>> Como a resposta é E (nenhuma das anteriores), não sei se é possível
>> provar que as anteriores são falsas. Eu não consegui concluir coisa alguma.
>>
>> *Seja f(x) = am.x^m + am–1.x^(m–1) + ... + a1.x + a0, onde am, am–1, ...,
>> a1, a0 são reais, am diferente de 0 e a0 diferente de 0. Se f(1) é solução
>> real da equação 2^(x–3) + 2^(x–4) = 2^(x–2) – 2^(x–1) + 14, f(–1) = 2.f(1)
>> e a0 = 2.f(–1), então podemos afirmar:*
>>
>> *a) f(x) tem somente raízes reais positivas.*
>>
>> *b) f(x) tem somente raízes reais negativas.*
>>
>> *c) f(x) tem somente raízes reais inteiras.*
>>
>> *d) f(x) não tem raízes reais inteiras.*
>>
>> *e) nda*
>> Alguém tem alguma ideia?
>> Muito obrigado!
>>
>> --
>> Esta mensagem foi verificada pelo sistema de antivírus e
>> acredita-se estar livre de perigo.
>
>
> --
> Esta mensagem foi verificada pelo sistema de antivírus e
> acredita-se estar livre de perigo.

-- 
Esta mensagem foi verificada pelo sistema de antiv�rus e
 acredita-se estar livre de perigo.



[obm-l] Questão do ITA

2018-10-10 Por tôpico Vanderlei Nemitz
Bom dia, pessoal!
Encontrei essa questão, que diz ser do ITA (eu particularmente não
encontrei na internet).
Como a resposta é E (nenhuma das anteriores), não sei se é possível provar
que as anteriores são falsas. Eu não consegui concluir coisa alguma.

*Seja f(x) = am.x^m + am–1.x^(m–1) + ... + a1.x + a0, onde am, am–1, ...,
a1, a0 são reais, am diferente de 0 e a0 diferente de 0. Se f(1) é solução
real da equação 2^(x–3) + 2^(x–4) = 2^(x–2) – 2^(x–1) + 14, f(–1) = 2.f(1)
e a0 = 2.f(–1), então podemos afirmar:*

*a) f(x) tem somente raízes reais positivas.*

*b) f(x) tem somente raízes reais negativas.*

*c) f(x) tem somente raízes reais inteiras.*

*d) f(x) não tem raízes reais inteiras.*

*e) nda*
Alguém tem alguma ideia?
Muito obrigado!

-- 
Esta mensagem foi verificada pelo sistema de antiv�rus e
 acredita-se estar livre de perigo.



Re: [obm-l] Desigualdade

2018-09-14 Por tôpico Vanderlei Nemitz
Muito obrigado, Claudio!
Vou analisar com calma suas contas, mas a ideia parece muito elegante!

Em qua, 12 de set de 2018 11:21, Claudio Buffara 
escreveu:

> Com certeza dá. A questão é saber se há alguma fórmula ou algoritmo
> engenhoso pra fazer isso sem "ir somando até passar de 1".
> Uma ideia é calcular uma cota inferior e uma cota superior pra soma
> 1/(n-k) + 1/(n-k+1) + ... + 1/n.
> Por exemplo, sabemos que:
> 1/(n-k) + ... + 1/(n-1) + 1/n < log(n) - log(n-k-1) < 1/(n-k-1) + ... +
> 1/(n-2) + 1/(n-1)
> (pra ver isso, faça o gráfico)
> Isso implica que 1/(n-k-1) + ... + 1/(n-1) + 1/n > log(n) - log(n-k-1) +
> 1/n.
> Assim, é suficiente (mas não necessário) achar k tal que:
> log(n) - log(n-k-1) <= 1 <= log(n) - log(n-k-1) + 1/n ==>
> log(n/(n-k-1)) <= 1 <= log(n/(n-k-1)) + 1/n ==>
> n/(n-k-1) <= e <= n/(n-k-1) * e^(1/n) ==>
> n*(1 - e^(1/n-1)) - 1 <= k <= n*(1 - e^(-1)) - 1
>
> Por exemplo, se n = 100, a desigualdade acima fica:
> 100*(1 - e^(-0,99)) - 1 <= k <= 100*(1 - e^(-1)) - 1
> 61,84 <= k <= 62,21 ==> k = 62.
> E, de fato, 1/38 + ... + 1/100 = 0,9858 e 1/37 + ... + 1/100 = 1,0128
>
> Já, se n = 200, as cotas acima serão 125,0553 e 125,4241, de modo que não
> há nenhum k (inteiro) no intervalo.
> No entanto, 1/75 + ... + 1/200 < 1 < 1/74 + ... + 1/200, de modo que o k
> desejado é 125.
>
> De qualquer forma, vale a fórmula k = parte inteira de n*(1 - e^(-1)) - 1,
> pelo menos pra n = 2 (k = 0) e pra n >= 4.
> Pra n = 3, k = 1 (1/2 + 1/3 < 1 < 1 + 1/2 + 1/3), mas a fórmula dá k = 0.
>
> Se eu não errei nenhuma conta, acho que é isso.
>
> []s,
> Claudio.
>
>
> On Wed, Sep 12, 2018 at 9:57 AM Vanderlei Nemitz 
> wrote:
>
>> Bom dia!
>> É possível determinar, em função de n, o maior valor de k tal que 1/n +
>> 1/(n - 1) + 1/(n - 2) + ... + 1/(n - k) < 1, em que n é um inteiro maior do
>> que 1?
>>
>> Muito obrigado!
>>
>> --
>> Esta mensagem foi verificada pelo sistema de antivírus e
>> acredita-se estar livre de perigo.
>
>
> --
> Esta mensagem foi verificada pelo sistema de antivírus e
> acredita-se estar livre de perigo.

-- 
Esta mensagem foi verificada pelo sistema de antiv�rus e
 acredita-se estar livre de perigo.



[obm-l] Desigualdade

2018-09-12 Por tôpico Vanderlei Nemitz
Bom dia!
É possível determinar, em função de n, o maior valor de k tal que 1/n +
1/(n - 1) + 1/(n - 2) + ... + 1/(n - k) < 1, em que n é um inteiro maior do
que 1?

Muito obrigado!

-- 
Esta mensagem foi verificada pelo sistema de antiv�rus e
 acredita-se estar livre de perigo.



[obm-l] Matrizes

2018-08-21 Por tôpico Vanderlei Nemitz
Boa noite, pessoal!
Resolvi a seguinte questão, mas de uma forma um tanto complicada.
Gostaria de uma solução mais simples.
Muito obrigado!
Vanderlei

*Sejam A e B matrizes reais n x n tais que AB + A + B = 0. Prove que AB =
BA.*

-- 
Esta mensagem foi verificada pelo sistema de antiv�rus e
 acredita-se estar livre de perigo.



Re: [obm-l] Mais uma de Geometria do IME

2018-07-19 Por tôpico Vanderlei Nemitz
Obrigado, Claudio!
Vou usar suas valiosas dicas para tentar resolver o problema!

Em qua, 18 de jul de 2018 11:51, Claudio Buffara 
escreveu:

> Mais uma observação...
>
> As três razões que entram na aplicação final do teorema de Menelaus
> recíproco ( AR/RB * BP/PC * CQ/QA = 1 ) devem ser expressas em termos de
> comprimentos de segmentos do triângulo ABC que, de alguma forma, terão que
> se cancelar (pro produto ser igual a 1).
>
> Assim, por exemplo, a expressão final de BP/PC não poderá ser em termos de
> DH e EH, pois estes dois segmentos só são relevantes em relação ao lado BC
> e à altura AH, mas não em relação ao lado AC e a correspondente altura BK
> (K em AC), e nem ao lado AB e altura CJ (J em AB).
>
> No entanto, o teorema de Ceva aplicado às alturas (que são concorrentes),
> implica que:
> AJ/JB * BH/HC * CK*KA = 1.
>
> Isso significa que talvez devêssemos procurar expressar a razão BP/PC em
> termos de BH e HC.
>
> []s,
> Claudio.
>
>
>
> 2018-07-18 10:59 GMT-03:00 Claudio Buffara :
>
>> A figura completa é chatinha de fazer e fica muito "entulhada".
>>
>> Mas também acho que Menelaus é o caminho.
>>
>> Nesse tipo de problema, eu diria que primeiro você precisa aplicar o
>> teorema de Menelaus direto (pontos colineares ==> produto das razões = -1)
>> a fim de achar as razões adequadas a partir de pontos que são sabidamente
>> colineares e, no final, pra provar que P, Q e R são colineares, você aplica
>> o recíproco (produto das razões = -1 ==> pontos colineares).
>>
>> Assim, começando pelo final, eu diria que o objetivo é provar que: AR/RB
>> * BP/PC * CQ/QA = -1 (ou, como não me parece que a orientação dos segmentos
>> é relevante nesse problema, basta trabalhar com os comprimentos sem sinal:  
>> AR/RB
>> * BP/PC * CQ/QA = 1)
>>
>> Como o enunciado fala em alturas e perpendiculares, vão aparecer vários
>> triângulos retângulos.
>> Isso significa que talvez seja necessário usar Pitágoras e também
>> semelhança (já que, por exemplo, os triângulos AHB, HDB e ADH são
>> semelhantes - com pontos correspondentes e, portanto, segmentos homólogos,
>> escritos na mesma ordem em cada um dos triângulos: sempre um bom hábito que
>> evita erros bobos).
>>
>> Por exemplo, os pontos mencionados no enunciado mostram o triângulo ABC
>> cortado pela reta PDE.
>> Menelaus aplicado a este triângulo e esta reta implica que: AD/DB * BP/PC
>> * CE/EA = 1 ==> BP/PC = DB/AD * EA/CE.
>>
>> Agora, DB e AD são lados dos triângulos semelhantes que eu mencionei
>> acima. Idem para EA e CE (neste caso, os triângulos semelhantes são AHC,
>> AEH e HEC).
>>
>> A semelhança de HDB e ADH implica que: DB/DH = DH/AD.
>> A semelhança de AEH e HEC implica que: EA/EH = EH/CE.
>> Infelizmente, isso resulta no produto DB*AD ao invés da razão AD/DB (idem
>> para EA*CE).
>>
>> E neste ponto eu empaquei...
>>
>> Mas acho que, como ainda não foram usados os triângulos AHB (semelhante a
>> HDB e ADH) e AHC (semelhante a AEH e HEC), que têm o lado AH comum, de
>> alguma forma (talvez usando Pitágoras) estes podem ajudar a encontrar
>> expressões úteis para as razões DB/AD e EA/CE, o que, por sua vez, nos
>> daria a razão BP/PC, a ser usada na aplicação final do recíproco do teorema
>> de Menelaus.
>>
>> Ou então, é claro, este caminho pode não levar a nada e será preciso
>> recomeçar do zero...matemática é assim mesmo...
>>
>> []s,
>> Claudio.
>>
>>
>> 2018-07-17 22:22 GMT-03:00 Vanderlei Nemitz :
>>
>>> A questão a seguir é da prova do IME de 1991. Tentei utilizar o teorema
>>> de Menelaus, mas não conseguir demonstrar. Como eu poderia fazer?
>>>
>>> Obrigado!
>>>
>>>
>>>
>>> Num triângulo ABC traçamos a altura AH e do pé H desta altura
>>> construímos as perpendiculares HD e HE sobre os lados AB e AC. Seja P o
>>> ponto da interseção de DE com BC. Construindo as alturas relativas aos
>>> vértices B e C determinam-se também, de modo análogo Q e R sobre os lados
>>> AC e AB. Demonstre que os pontos P, Q, R são colineares.
>>>
>>>
>>> --
>>> Esta mensagem foi verificada pelo sistema de antivírus e
>>> acredita-se estar livre de perigo.
>>
>>
>>
>
> --
> Esta mensagem foi verificada pelo sistema de antivírus e
> acredita-se estar livre de perigo.

-- 
Esta mensagem foi verificada pelo sistema de antiv�rus e
 acredita-se estar livre de perigo.



[obm-l] Mais uma de Geometria do IME

2018-07-17 Por tôpico Vanderlei Nemitz
A questão a seguir é da prova do IME de 1991. Tentei utilizar o teorema de
Menelaus, mas não conseguir demonstrar. Como eu poderia fazer?

Obrigado!



Num triângulo ABC traçamos a altura AH e do pé H desta altura construímos
as perpendiculares HD e HE sobre os lados AB e AC. Seja P o ponto da
interseção de DE com BC. Construindo as alturas relativas aos vértices B e
C determinam-se também, de modo análogo Q e R sobre os lados AC e AB.
Demonstre que os pontos P, Q, R são colineares.

-- 
Esta mensagem foi verificada pelo sistema de antiv�rus e
 acredita-se estar livre de perigo.



[obm-l] Re: [obm-l] Re: [obm-l] Questão do IME

2018-07-14 Por tôpico Vanderlei Nemitz
Muito obrigado, Claudio!
Bela solução!

Em 13 de julho de 2018 13:35, Claudio Buffara 
escreveu:

> Os prolongamentos de DM e EN se intersectam num mesmo ponto P pertencente
> a AB.
> Pra ver isso, repare que os triângulos DCM e PAM são semelhantes (razão de
> semelhança = 2).
> Idem para os triângulos EFN e PNB.
> Como, no triângulo PDE (que é isósceles), vale PM/PD = PN/PE = 1/3,
> concluímos que MN é paralelo a DE.
>
> []s,
> Claudio.
>
>
> 2018-07-13 12:13 GMT-03:00 Vanderlei Nemitz :
>
>> Sejam dois quadrados ABCD e ABEF, tendo um lado comum AB, mas não
>> situados num mesmo plano. Sejam M e N pertencentes, respectivamente, às
>> diagonais AC e BF tais que AM/AC = BN/BF = 1/3. Mostre que MN é paralelo a
>> DE.
>>
>> Alguém poderia ajudar?
>> Obrigado,
>> Vanderlei
>>
>> --
>> Esta mensagem foi verificada pelo sistema de antivírus e
>> acredita-se estar livre de perigo.
>
>
>
> --
> Esta mensagem foi verificada pelo sistema de antivírus e
> acredita-se estar livre de perigo.

-- 
Esta mensagem foi verificada pelo sistema de antiv�rus e
 acredita-se estar livre de perigo.



[obm-l] Questão do IME

2018-07-13 Por tôpico Vanderlei Nemitz
Sejam dois quadrados ABCD e ABEF, tendo um lado comum AB, mas não situados
num mesmo plano. Sejam M e N pertencentes, respectivamente, às diagonais AC
e BF tais que AM/AC = BN/BF = 1/3. Mostre que MN é paralelo a DE.

Alguém poderia ajudar?
Obrigado,
Vanderlei

-- 
Esta mensagem foi verificada pelo sistema de antiv�rus e
 acredita-se estar livre de perigo.



[obm-l] Problema da OBM 2017

2018-05-10 Por tôpico Vanderlei Nemitz
Pessoal, gostaria de uma ajuda no item c dessa questão. Os dois primeiros
itens são tranquilos.

*Na Terra dos Impas, somente os algarismos ímpares são utilizados para
contar e escrever números. Assim, em vez dos números 1, 2, 3, 4, 5, 6, 7,
8, 9, 10, 11, 12, . . . os Impas tem os números correspondentes 1, 3, 5, 7,
9, 11, 13, 15, 17, 19, 31, 33, . . . (note que os números dos Impas têm
somente algarismos ímpares). Por exemplo, se uma criança tem 11 anos, os
Impas diriam que ela tem 31 anos. *

*a) Como os Impas escrevem o nosso numero 20? *

*b) Numa escola desse lugar, a professora escreveu no quadro-negro a
continha de multiplicar abaixo. Se você fosse um aluno Impa, o que
escreveria como resultado?*

*13 × 5*

*c) Escreva, na linguagem dos Impas, o numero que na nossa representação ao
decimal é escrito como 2017.*

Obrigado!

-- 
Esta mensagem foi verificada pelo sistema de antiv�rus e
 acredita-se estar livre de perigo.



Re: [obm-l] Limite

2018-03-19 Por tôpico Vanderlei Nemitz
Obrigado! Mesmo assim, se alguém puder postar a resolução...

Em seg, 19 de mar de 2018 13:09, Bernardo Freitas Paulo da Costa <
bernardo...@gmail.com> escreveu:

> 2018-03-19 12:27 GMT-03:00 Vanderlei Nemitz <vanderma...@gmail.com>:
> > Bom dia!
> > Eu resolvi o limite a seguir de um modo muito complicado e encontrei 1/e.
> >
> > Alguém conhece alguma solução?
> >
> > lim [n!/n^n]^(1/n), quando n tende ao infinito.
>
> Eu imagino que seja para usar a equivalência entre o teste da raiz e o
> teste da razão.
>
> Abraços,
> --
> Bernardo Freitas Paulo da Costa
>
> --
> Esta mensagem foi verificada pelo sistema de antivírus e
>  acredita-se estar livre de perigo.
>
>
> =
> Instruções para entrar na lista, sair da lista e usar a lista em
> http://www.mat.puc-rio.br/~obmlistas/obm-l.html
> =
>

-- 
Esta mensagem foi verificada pelo sistema de antiv�rus e
 acredita-se estar livre de perigo.



[obm-l] Limite

2018-03-19 Por tôpico Vanderlei Nemitz
Bom dia!
Eu resolvi o limite a seguir de um modo muito complicado e encontrei 1/e.

Alguém conhece alguma solução?

lim [n!/n^n]^(1/n), quando n tende ao infinito.

Muito obrigado!

-- 
Esta mensagem foi verificada pelo sistema de antiv�rus e
 acredita-se estar livre de perigo.



Re: [obm-l] Dica Sobre Livros

2017-07-29 Por tôpico Vanderlei Nemitz
Eu também gostaria!

vanderma...@gmail.com

Muito obrigado,

Vanderlei

Em 29 de julho de 2017 15:06, Tássio Naia  escreveu:

> Alguém falou para mim do libgen... talvez ajude?
>
> Até+
>
> On Sat, Jul 29, 2017 at 3:52 PM, Ricardo Leão 
> wrote:
>
>> Eu tenho procurado os seguintes livros:
>>
>> - Andreescu, T; Kedlaya, K; Zeitz, P; *Mathematical Contests 1995-1996:
>> Olympiad Problems from around the world, with solutions* (1997)
>>
>> - Andreescu, T; Kedlaya, K; *Mathematical Contests 1996-1997: Olympiad
>> Problems from around the world, with solutions* (1998)
>>
>> - Andreescu, T; Kedlaya, K; *Mathematical Contests 1997-1998: Olympiad
>> Problems from around the world, with solutions* (1999)
>>
>> Alguém aí sabe onde eu encontro esses livros em formato físico ou
>> digital???
>>
>> --
>> Esta mensagem foi verificada pelo sistema de antivírus e
>> acredita-se estar livre de perigo.
>
>
>
> --
> Esta mensagem foi verificada pelo sistema de antivírus e
> acredita-se estar livre de perigo.
>

-- 
Esta mensagem foi verificada pelo sistema de antiv�rus e
 acredita-se estar livre de perigo.



Re: [obm-l] Somas iguais

2017-07-09 Por tôpico Vanderlei Nemitz
Obrigado, Pedro!
Acho que ficou claro, sim!

Em 8 de jul de 2017 3:51 PM, "Pedro Soares" <pedrosoares...@gmail.com>
escreveu:

> Desculpe se ficou mal escrito* heheh
>
>
> <http://www.avg.com/email-signature?utm_medium=email_source=link_campaign=sig-email_content=webmail>
>  Virus-free.
> www.avg.com
> <http://www.avg.com/email-signature?utm_medium=email_source=link_campaign=sig-email_content=webmail>
> <#m_-8671497293299101645_DAB4FAD8-2DD7-40BB-A1B8-4E2AA1F9FDF2>
>
> 2017-07-08 15:26 GMT-03:00 Pedro Soares <pedrosoares...@gmail.com>:
>
>> Para a soma de n números naturais ser par essa sequência deve possuir um
>> número par de números impares. Logo, se está se somando de 1 a n e a soma é
>> par  para n = 2k - 1 ou n = 2k onde k é multiplo de 2( se k for impar
>> teremos um número impar de números impares na soma).
>> O caso em que n=2k é trivial, pode-se pegar os extremos da soma e colocar
>> em um subgrupo, os próximos extremos colocar no outro subgrupo e repetir
>> essa ação k/2 vezes( lembre-se que k é multiplo de 2, então podemos fazer
>> isso).
>> Para n = 2k - 1 primeiro olhe para k = 2, claramente podemos separar nos
>> subgrupos {1,2} e {3} que possuem a mesma soma.
>> Agora suponha que vale para k = j, vamos provar que vale para k = n + 2
>> por indução.
>> A soma para n = 2( k + 2 ) + 1 é igual a soma para n = 2k( que vamos
>> chamar de S(n) ) mais quatro termos consecutivos ( n+1, n+2, n+3, n+4).
>> S(n) já sabemos dividir em subgrupos de igual soma por hipótese. Além
>> disso, podemos alocar os termos faltantes usando a mesma estratégia usada
>> para o caso n=2k( os termos n+1 e n+4 vão para um subgrupo e os termos n+2
>> e n+3 vão para o outro). Logo, se vale para k = j vale k = j + 2. Como vale
>> para k = 2 vale para todo multiplo de 2.
>> Como já provamos para os dois casos em que separamos isso conclui nossa
>> prova :)
>>
>> Desculpe se ficou mau escrito, digitei conforme fui pensando
>>
>>
>> On Saturday, 8 July 2017, Vanderlei Nemitz <vanderma...@gmail.com> wrote:
>>
>>> Bom dia!
>>> Gostaria de saber se alguém tem uma solução para esse problema:
>>>
>>> *Mostre que se a soma dos números de 1 até n é par, então é possível
>>> separar os números de 1 até n em dois subgrupos de números de igual soma.*
>>>
>>> Muito obrigado!
>>>
>>> Vanderlei
>>>
>>> --
>>> Esta mensagem foi verificada pelo sistema de antivírus e
>>> acredita-se estar livre de perigo.
>>
>>
>
> --
> Esta mensagem foi verificada pelo sistema de antivírus e
> acredita-se estar livre de perigo.

-- 
Esta mensagem foi verificada pelo sistema de antiv�rus e
 acredita-se estar livre de perigo.



[obm-l] Somas iguais

2017-07-08 Por tôpico Vanderlei Nemitz
Bom dia!
Gostaria de saber se alguém tem uma solução para esse problema:

*Mostre que se a soma dos números de 1 até n é par, então é possível
separar os números de 1 até n em dois subgrupos de números de igual soma.*

Muito obrigado!

Vanderlei

-- 
Esta mensagem foi verificada pelo sistema de antiv�rus e
 acredita-se estar livre de perigo.



[obm-l] Re: [obm-l] Re: [obm-l] Polinômios

2017-05-27 Por tôpico Vanderlei Nemitz
Muito obrigado, Douglas!

Eu não conhecia esse teorema. Com certeza é muito valioso!

Em 27 de maio de 2017 17:08, Douglas Oliveira de Lima <
profdouglaso.del...@gmail.com> escreveu:

> Então:
>
> *Podemos usar o seguinte teorema: Na divisão de um polinômio p(x) por
> h1(x) o resto é r1(x); na divisão de p(x) por h2(x) o resto é r2(x); na
> divisão de p(x) por h1(x).h2(x) o resto é r(x). Se r(x) é dividido por
> h1(x) o resto é r1(x) e dividido por h2(x) o resto é r2(x).*
>
> *O resto da divisão de P(x) por x4 + x2 + 1  possui de grau menor ou igual
> a 3:   r(x) = ax3 + bx2 + cx + d*
>
> *De acordo com o teorema,  ax3 + bx2 + cx + d  dividido por  x2 + x + 1
> deixa resto – x + 1  e  dividido por x2 – x + 1  deixa resto 3x + 5.
> Então:   i)  ax3 + bx2 + cx + d = (x2 + x + 1)(ax + e) – x + 1   =>*
>
> *ax3 + bx2 + cx + d = ax3 + (a + e)x2 + (a + e – 1)x + e + 1*
>
> *ii) ax3 + bx2 + cx + d = (x2 – x + 1)(ax + f) + 3x + 5   =>*
>
> *ax3 + bx2 + cx + d = ax3 + (f – a)x2 + (a – f + 3)x + f + 5*
>
> *\**e + 1 = f + 5   =>   e – f = 4 **\**a + e – 1 = a – f + 3   =>
> e + f = 4   =>  e = 4   e   f = 0*
>
> *\**d = e + 1   =>   d = 5 **\**a + e = f – a   =>   2a = – 4   =>
> a = – 2 **\**b = f – a   =>   b = 2*
>
> *\**c = a + e – 1 = – 2 + 4 – 1   =>   c = 1**\**Ou seja:   r(x)
> = – 2x3 + 2x2 + x + 5*
>
>
> *Observação: O que fiz nada mais foi do que congruência aplicada a
> polinômios.*
>
>
> *Abraços *
>
> *Douglas Oliveira*
>
> Em 27 de maio de 2017 11:17, Vanderlei Nemitz <vanderma...@gmail.com>
> escreveu:
>
>> Bom dia!
>>
>> Alguém poderia dar uma ideia na seguinte questão? Já tentes algumas
>> estratégias, mas sem êxito.
>>
>> Um polinômio P(x) dividido por x^2 + x + 1 dá resto -x + 1 e dividido por
>> x^2 -x + 1 dá resto 3x + 5. Qual o resto da divisão de P(x) por x^4 + x^2 +
>> 1?
>>
>> A resposta que tenho é -2x^3 + 2x^2 + x + 5.
>>
>> Obrigado!
>>
>> Vanderlei
>>
>> --
>> Esta mensagem foi verificada pelo sistema de antivírus e
>> acredita-se estar livre de perigo.
>
>
>
> --
> Esta mensagem foi verificada pelo sistema de antivírus e
> acredita-se estar livre de perigo.
>

-- 
Esta mensagem foi verificada pelo sistema de antiv�rus e
 acredita-se estar livre de perigo.



[obm-l] Polinômios

2017-05-27 Por tôpico Vanderlei Nemitz
Bom dia!

Alguém poderia dar uma ideia na seguinte questão? Já tentes algumas
estratégias, mas sem êxito.

*Um polinômio P(x) dividido por x^2 + x + 1 dá resto -x + 1 e dividido por
x^2 -x + 1 dá resto 3x + 5. Qual o resto da divisão de P(x) por x^4 + x^2 +
1?*

A resposta que tenho é *-2x^3 + 2x^2 + x + 5*.

Obrigado!

Vanderlei

-- 
Esta mensagem foi verificada pelo sistema de antiv�rus e
 acredita-se estar livre de perigo.



[obm-l] Questão de um vestibular do Acre

2016-12-21 Por tôpico Vanderlei Nemitz
Boa tarde!
Tentei resolver uma questão de um vestibular do Acre, mas parece que faltam
informações, que talvez seja necessário supor.
Como acho que não posso anexar um arquivo aqui, deixo um link que acessa a
prova. É a questão *32*, de geometria plana.

http://www.strixeducacao.com.br/vs-arquivos/HtmlEditor/file/PROVAS%20APLICADAS/Uninorte_2016_2_Tipo1.pdf

Muito obrigado!

Vanderlei

-- 
Esta mensagem foi verificada pelo sistema de antiv�rus e
 acredita-se estar livre de perigo.



[obm-l] Re: [obm-l] Re: [obm-l] Combinatória

2016-06-15 Por tôpico Vanderlei Nemitz
Maurício:

Aparentemente sua solução está perfeita. Agradeço muito! Ficou bem elegante!

Um abraço!

Vanderlei

Em 14 de junho de 2016 21:02, Mauricio de Araujo <
mauricio.de.ara...@gmail.com> escreveu:

> Para deixar claro a questão da divisão por dois:
>
> Nossa estratégia para montar uma comissão "não válida" é escolher um
> senador entre os 30, depois escolher um inimigo e depois escolher um amigo.
>
> Imagine que escolhemos inicialmente o senador A para formar a comissão
> {A,C,B} onde A é amigo de B e inimigo de C... Esta mesma comissão é
> escolhida novamente porque uma das duas situações abaixo necessariamente
> acontece...
>
> Se B for amigo de C, a mesma comissão aparecerá quando a escolha começar
> pelo senador C onde a comissão será {C,A,B}
>
> Se B for inimigo de C, a mesma comissão aparecerá quando a escolha começar
> pelo senador B onde a comissão será {B,C,A}.
>
> Logo, a mesma comissão é contada duas vezes...
>
>
>
> Em 14 de junho de 2016 20:17, Mauricio de Araujo <
> mauricio.de.ara...@gmail.com> escreveu:
>
>> Ataquemos o problema olhando o contrário do que se quer, ou seja, vendo
>> as comissões onde haja um amigo e um inimigo de um senador em particular...
>>
>> Isso pode ser feito assim:
>>
>> Número de escolhas de um certo senador: 30
>> Número de inimigos a escolher para compor a comissão: 6
>> Número de amigos a escolher para compor a comissão: 23
>>
>> Logo o total de comissões onde há um amigo e um inimigo de um certo
>> senador é: 30.6.23 = 4140. Entretanto, temos de dividir este número por
>> dois porque a mesma comissão aparece quando o senador escolhido é o amigo
>> do primeiro... Logo o total de comissões onde existe "amigos e inimigos" é
>> 2070.
>>
>> O total de comissões é igual a C30,3 = 4060.
>>
>> Logo o que se quer é 4060 - 2070 = 1990.
>>
>> Acho que é isso
>>
>> Em 11 de junho de 2016 17:21, Vanderlei Nemitz <vanderma...@gmail.com>
>> escreveu:
>>
>>> Gostaria de uma ajuda para o seguinte problema.
>>> A resposta é 1990
>>> Obrigado!
>>>
>>>
>>> Em um senado, há 30 senadores. Para cada par de senadores, eles podem
>>> ser amigos ou inimigos. Cada senador tem 6 inimigos. Considere comissões
>>> formadas por 3 senadores. Determine o número total de comissões, cujos
>>> membros são todos amigos uns dos outros ou todos inimigos uns dos outros.
>>>
>>>
>>>
>>> --
>>> Esta mensagem foi verificada pelo sistema de antivírus e
>>> acredita-se estar livre de perigo.
>>
>>
>>
>>
>> --
>>
>> Abraços,
>> oɾnɐɹɐ ǝp oıɔıɹnɐɯ
>>
>>
>
>
> --
>
> Abraços,
> oɾnɐɹɐ ǝp oıɔıɹnɐɯ
>
>
> --
> Esta mensagem foi verificada pelo sistema de antivírus e
> acredita-se estar livre de perigo.
>

-- 
Esta mensagem foi verificada pelo sistema de antiv�rus e
 acredita-se estar livre de perigo.



[obm-l] Combinatória

2016-06-11 Por tôpico Vanderlei Nemitz
Gostaria de uma ajuda para o seguinte problema.
A resposta é 1990
Obrigado!


Em um senado, há 30 senadores. Para cada par de senadores, eles podem ser
amigos ou inimigos. Cada senador tem 6 inimigos. Considere comissões
formadas por 3 senadores. Determine o número total de comissões, cujos
membros são todos amigos uns dos outros ou todos inimigos uns dos outros.

-- 
Esta mensagem foi verificada pelo sistema de antiv�rus e
 acredita-se estar livre de perigo.



[obm-l] Função

2016-04-05 Por tôpico Vanderlei Nemitz
Oi, pessoal, tudo bem? Gostaria de saber se alguém consegue resolver a
seguinte questão. O que eu gostaria é "provar" genericamente e não concluir
qual é a alternativa correta usando exemplos numéricos, pois isso é
simples! Muito obrigado!

Para *x* e *y* inteiros estritamente positivos, considere a função:

F(x, y) = F(x – y, y), se x > y

F(x, y) = F(x, y – x), se x < y

F(x, y) = x, se x = y

Podemos concluir que

a) F(x, y) = 1 para quaisquer x e y

b) F(x, y) = 2 se x for múltiplo de y

c) F(x, y) = mdc(x, y) para quaisquer x e y

d) F(x, y) = mmc(x, y) para quaisquer x e y

e) F(x, y) = 1 se x for um número primo

-- 
Esta mensagem foi verificada pelo sistema de antiv�rus e
 acredita-se estar livre de perigo.



Re: [obm-l] Livros

2016-01-18 Por tôpico Vanderlei Nemitz
Muito obrigado!!!

Em 18 de janeiro de 2016 22:20, Mauricio de Araujo <
mauricio.de.ara...@gmail.com> escreveu:

> Tenho eles em russo... vou deixar no link abaixo por 2 dias.
>
> https://drive.google.com/folderview?id=0B-1sAhj7LSlyd3UzcTNSOWdjdzg=sharing
>
> Em 18 de janeiro de 2016 18:48, regis barros <regisgbar...@yahoo.com.br>
> escreveu:
>
>> Boa noite Pessoal
>> Verifiquei meu hd se há algum livro do suprun, mas lamento não há nenhum.
>>
>> Regis
>>
>>
>> Em Segunda-feira, 18 de Janeiro de 2016 12:23, Guilherme Ribeiro <
>> gprfaw...@gmail.com> escreveu:
>>
>>
>> Também quero!
>> Em 18/01/2016 03:28, "Research" <mathemat...@sapo.pt> escreveu:
>>
>> Prezados,
>> Faço minhas as palavras do Eduardo Beltrão.
>> mathemat...@sapo.pt
>>
>> Atenciosamente,
>>
>> Nzinga
>>
>>
>> On Jan 16, 28 Heisei, at 11:03 AM, e-...@ig.com.br wrote:
>>
>> Se alguém disponibilizar tais obras em PDF também gostaria de uma cópia.
>> Se no DROPBOX, melhor ainda. Se não, segue meu e-mail:
>> e-...@ig.com.br
>> Desde já, agradeço!
>> Eduardo Beltrão
>>
>>
>> Em 14/01/2016 21:14, Jeferson Almir escreveu:
>>
>> Reintero o meu interesse por esses livros, caso alguém já obteve poderia
>> disponibilizar uma pasta compartilhada no Dropbox seria uma boa ideia.
>> Abraço Jeferson Almir
>>
>> Em quinta-feira, 14 de janeiro de 2016, Giovanni Celestre <
>> ggabrie...@gmail.com> escreveu:
>>
>> Eu também, Por favor
>> Obrigado
>>
>> 2016-01-14 13:01 GMT-02:00 Vanderlei Nemitz <vanderma...@gmail.com>:
>>
>> Eu quero, Israel!
>>
>> Obrigado!
>>
>> Vanderlei
>>
>> Em 14 de janeiro de 2016 12:54, Israel Meireles Chrisostomo <
>> israelmchrisost...@gmail.com> escreveu:
>>
>> *PROBLEMAS DE ALTA DIFICULDAD - 300 Problemas Resolvidos*
>>
>> *Métodos Alternativos para a Resolução de Equações e Inequações - 350
>> Problemas Resolvidos*Eu tenho esses doi em espanhol, não é pdf, é o
>> livro mesmo!Se tiver interessado mande um email para mim!
>>
>> Em 14 de janeiro de 2016 11:07, benedito freire <bened...@ufrnet.br>
>> escreveu:
>>
>> Por favor, escreva o nome do autor completo.Talvez eu
>> possa conseguir
>> ------
>> De: Vanderlei Nemitz
>> Enviada em: ‎14/‎01/‎2016 11:12
>> Para: OBM
>> Assunto: Re: [obm-l] Livros
>>
>> Você tem algum deles, Regis? Eu tinha o PDF de dois deles, em Russo, mas
>> o pendrive estragou e perdi :(
>>
>> Em 14 de janeiro de 2016 11:01, Jefferson Cândido <jjjeffer...@gmail.com>
>> escreveu:
>>
>> Muito bom! Se puder mandar também para meu e-mail, jjjeffer...@gmail.com,
>> agradeço!
>>
>> Em 13 de janeiro de 2016 21:45, Vanderlei Nemitz <vanderma...@gmail.com>
>> escreveu:
>>
>> *PROBLEMAS DE ALTA DIFICULDAD - 300 Problemas Resolvidos*
>> *Métodos de Resoluções e Demonstrações de Desigualdades - ** 367
>> Problemas*
>> *Métodos Alternativos para a Resolução de Equações e Inequações - 350
>> Problemas Resolvidos*
>>
>> *Qualquer um desses já seria uma grande ajuda!*
>>
>> *Obrigado!*
>>
>> Em 13 de janeiro de 2016 21:33, regis barros <regisgbar...@yahoo.com.br>
>> escreveu:
>>
>> Olá Vanderlei
>> Quais livros do suprun você precisa?
>>
>> Regis
>>
>>
>> Em Quarta-feira, 13 de Janeiro de 2016 14:35, Vanderlei Nemitz <
>> vanderma...@gmail.com> escreveu:
>>
>>
>> Boa tarde! Alguém tem os PDFs dos livros do Suprún? Pode ser até em russo
>> mesmo! Ou mesmo tenha e queira vender os livros físicos? Preciso muito
>> deles, mas está em falta.
>>
>> Obrigado!
>>
>>
>>
>>
>>
>> --
>> É preciso amar as pessoas como se não houvesse amanhã...
>>
>> Jefferson Cândido -
>>
>>
>>
>>
>>
>
>
> --
> Abraços
>
> oɾnɐɹɐ ǝp oıɔıɹnɐɯ
>
>


Re: [obm-l] Livros

2016-01-14 Por tôpico Vanderlei Nemitz
Você tem algum deles, Regis? Eu tinha o PDF de dois deles, em Russo, mas o
pendrive estragou e perdi :(

Em 14 de janeiro de 2016 11:01, Jefferson Cândido <jjjeffer...@gmail.com>
escreveu:

> Muito bom! Se puder mandar também para meu e-mail, jjjeffer...@gmail.com,
> agradeço!
>
> Em 13 de janeiro de 2016 21:45, Vanderlei Nemitz <vanderma...@gmail.com>
> escreveu:
>
>> *PROBLEMAS DE ALTA DIFICULDAD - 300 Problemas Resolvidos*
>> *Métodos de Resoluções e Demonstrações de Desigualdades - ** 367
>> Problemas*
>> *Métodos Alternativos para a Resolução de Equações e Inequações - 350
>> Problemas Resolvidos*
>>
>> *Qualquer um desses já seria uma grande ajuda!*
>>
>> *Obrigado!*
>>
>> Em 13 de janeiro de 2016 21:33, regis barros <regisgbar...@yahoo.com.br>
>> escreveu:
>>
>>> Olá Vanderlei
>>> Quais livros do suprun você precisa?
>>>
>>> Regis
>>>
>>>
>>> Em Quarta-feira, 13 de Janeiro de 2016 14:35, Vanderlei Nemitz <
>>> vanderma...@gmail.com> escreveu:
>>>
>>>
>>> Boa tarde! Alguém tem os PDFs dos livros do Suprún? Pode ser até em
>>> russo mesmo! Ou mesmo tenha e queira vender os livros físicos? Preciso
>>> muito deles, mas está em falta.
>>>
>>> Obrigado!
>>>
>>>
>>>
>>
>
>
> --
> É preciso amar as pessoas como se não houvesse amanhã...
>
> Jefferson Cândido -
>


Re: [obm-l] Livros

2016-01-14 Por tôpico Vanderlei Nemitz
Eu quero, Israel!

Obrigado!

Vanderlei

Em 14 de janeiro de 2016 12:54, Israel Meireles Chrisostomo <
israelmchrisost...@gmail.com> escreveu:

> *PROBLEMAS DE ALTA DIFICULDAD - 300 Problemas Resolvidos*
>
> *Métodos Alternativos para a Resolução de Equações e Inequações - 350
> Problemas Resolvidos*Eu tenho esses doi em espanhol, não é pdf, é o livro
> mesmo!Se tiver interessado mande um email para mim!
>
> Em 14 de janeiro de 2016 11:07, benedito freire <bened...@ufrnet.br>
> escreveu:
>
>> Por favor, escreva o nome do autor completo.Talvez eu
>> possa conseguir
>> --
>> De: Vanderlei Nemitz <vanderma...@gmail.com>
>> Enviada em: ‎14/‎01/‎2016 11:12
>> Para: OBM <obm-l@mat.puc-rio.br>
>> Assunto: Re: [obm-l] Livros
>>
>> Você tem algum deles, Regis? Eu tinha o PDF de dois deles, em Russo, mas
>> o pendrive estragou e perdi :(
>>
>> Em 14 de janeiro de 2016 11:01, Jefferson Cândido <jjjeffer...@gmail.com>
>> escreveu:
>>
>>> Muito bom! Se puder mandar também para meu e-mail, jjjeffer...@gmail.com,
>>> agradeço!
>>>
>>> Em 13 de janeiro de 2016 21:45, Vanderlei Nemitz <vanderma...@gmail.com>
>>> escreveu:
>>>
>>>> *PROBLEMAS DE ALTA DIFICULDAD - 300 Problemas Resolvidos*
>>>> *Métodos de Resoluções e Demonstrações de Desigualdades - ** 367
>>>> Problemas*
>>>> *Métodos Alternativos para a Resolução de Equações e Inequações - 350
>>>> Problemas Resolvidos*
>>>>
>>>> *Qualquer um desses já seria uma grande ajuda!*
>>>>
>>>> *Obrigado!*
>>>>
>>>> Em 13 de janeiro de 2016 21:33, regis barros <regisgbar...@yahoo.com.br
>>>> > escreveu:
>>>>
>>>>> Olá Vanderlei
>>>>> Quais livros do suprun você precisa?
>>>>>
>>>>> Regis
>>>>>
>>>>>
>>>>> Em Quarta-feira, 13 de Janeiro de 2016 14:35, Vanderlei Nemitz <
>>>>> vanderma...@gmail.com> escreveu:
>>>>>
>>>>>
>>>>> Boa tarde! Alguém tem os PDFs dos livros do Suprún? Pode ser até em
>>>>> russo mesmo! Ou mesmo tenha e queira vender os livros físicos? Preciso
>>>>> muito deles, mas está em falta.
>>>>>
>>>>> Obrigado!
>>>>>
>>>>>
>>>>>
>>>>
>>>
>>>
>>> --
>>> É preciso amar as pessoas como se não houvesse amanhã...
>>>
>>> Jefferson Cândido -
>>>
>>
>>
>


Re: [obm-l] Livros

2016-01-13 Por tôpico Vanderlei Nemitz
*PROBLEMAS DE ALTA DIFICULDAD - 300 Problemas Resolvidos*
*Métodos de Resoluções e Demonstrações de Desigualdades - ** 367 Problemas*
*Métodos Alternativos para a Resolução de Equações e Inequações - 350
Problemas Resolvidos*

*Qualquer um desses já seria uma grande ajuda!*

*Obrigado!*

Em 13 de janeiro de 2016 21:33, regis barros <regisgbar...@yahoo.com.br>
escreveu:

> Olá Vanderlei
> Quais livros do suprun você precisa?
>
> Regis
>
>
> Em Quarta-feira, 13 de Janeiro de 2016 14:35, Vanderlei Nemitz <
> vanderma...@gmail.com> escreveu:
>
>
> Boa tarde! Alguém tem os PDFs dos livros do Suprún? Pode ser até em russo
> mesmo! Ou mesmo tenha e queira vender os livros físicos? Preciso muito
> deles, mas está em falta.
>
> Obrigado!
>
>
>


[obm-l] Livros

2016-01-13 Por tôpico Vanderlei Nemitz
Boa tarde! Alguém tem os PDFs dos livros do Suprún? Pode ser até em russo
mesmo! Ou mesmo tenha e queira vender os livros físicos? Preciso muito
deles, mas está em falta.

Obrigado!


[obm-l] Re: [obm-l] Análise combinatória

2015-12-10 Por tôpico Vanderlei Nemitz
Gabriel:
É justamente esse último 5! que eu tenho dúvidas. A permutação é circular,
certo? Mesmo assim multiplicamos por 5!? Sim, percebi o erro de digitação,
mas isso não é o principal.

Em 10 de dezembro de 2015 17:23, Gabriel Tostes <gtos...@icloud.com>
escreveu:

> A respostas 45360 está correta... Numere as cadeiras de 1 a 15 e dívida em
> 3 em casos:
> 1-> 15 ocupada
> 2-> 1 ocupada (análogo ao 1º)
> 3-> 1 e 15 vazias.
>
> No primeiro caso temos que 1 e 14 devem estar vazias, logo, temos 4
> pessoas para distribuir nas 12 cadeiras restantes...
> Como cada pessoa deve ocupar uma dessas cadeiras restam 8 vazias pra
> distribuir entre as pessoas, mas entre duas pessoas deve ter ao menos
> cadeira vazia, então -> 9!/5!x4!=136
> No terceiro caso temos 13 cadeiras pra colocar 5 pessoas, logo 8 vazias,
> mas entre duas pessoas devemos ter uma cadeira vazia pelo menos uma vazia
> -> 9!/4!x5!=136
> Total-> (2x136+136)x5!=45360
>
> On Dec 10, 2015, at 16:45, Vanderlei Nemitz <vanderma...@gmail.com> wrote:
>
> Pessoal, gostaria de uma ajuda com essa questão. Vi em um site a resposta
> 45360, mas não concordo. Encontrei um valor bem menor. Obrigado!
>
> Vanderlei
>
> *Cinco pessoas devem se sentar em 15 cadeiras colocadas em torno de uma
> mesa circular. De quantos modos isso pode ser feito se não deve haver
> ocupação simultânea de duas cadeiras adjacentes? *
>
> --
> Esta mensagem foi verificada pelo sistema de antivírus e
> acredita-se estar livre de perigo.
>
>
> --
> Esta mensagem foi verificada pelo sistema de antivírus e
> acredita-se estar livre de perigo.
>

-- 
Esta mensagem foi verificada pelo sistema de antiv�rus e
 acredita-se estar livre de perigo.



[obm-l] Análise combinatória

2015-12-10 Por tôpico Vanderlei Nemitz
Pessoal, gostaria de uma ajuda com essa questão. Vi em um site a resposta
45360, mas não concordo. Encontrei um valor bem menor. Obrigado!

Vanderlei

*Cinco pessoas devem se sentar em 15 cadeiras colocadas em torno de uma
mesa circular. De quantos modos isso pode ser feito se não deve haver
ocupação simultânea de duas cadeiras adjacentes? *

-- 
Esta mensagem foi verificada pelo sistema de antiv�rus e
 acredita-se estar livre de perigo.



[obm-l] Re: [obm-l] Re: [obm-l] Análise combinatória

2015-12-10 Por tôpico Vanderlei Nemitz
Mas então é levado em consideração a posição relativa das pessoas e das
cadeiras vazias? Por exemplo, se um pessoa A está nas mesmas posições
relativas em relação às pessoas B, C, D, E, mas ao seu lados estão outras
cadeiras vazias, a distribuição é considerada diferente? Pois caso não
seja, pensei que deveríamos multiplicar por (5 - 1)! = 24. Claro que meu
raciocínio pode estar falho!

Em 10 de dezembro de 2015 17:45, Gabriel Tostes <gtos...@icloud.com>
escreveu:

> Sim... Dividi em casos pra "tirar" a permutacao circular. O 136 de cada
> caso significa 136 modos de organizar as Cadeiras em "vazias" e "com
> Pessoas". Temos 5! Maneiras de distribuir as Pessoas nelas.
> On Dec 10, 2015, at 17:34, Vanderlei Nemitz <vanderma...@gmail.com> wrote:
>
> Gabriel:
> É justamente esse último 5! que eu tenho dúvidas. A permutação é
> circular, certo? Mesmo assim multiplicamos por 5!? Sim, percebi o erro de
> digitação, mas isso não é o principal.Â
>
> Em 10 de dezembro de 2015 17:23, Gabriel Tostes <gtos...@icloud.com>
> escreveu:
>
>> A respostas 45360 está correta... Numere as cadeiras de 1 a 15 e dívida
>> em 3 em casos:Â
>> 1-> 15 ocupada
>> 2-> 1 ocupada (análogo ao 1º)
>> 3-> 1 e 15 vazias.
>>
>> No primeiro caso temos que 1 e 14 devem estar vazias, logo, temos 4
>> pessoas para distribuir nas 12 cadeiras restantes...Â
>> Como cada pessoa deve ocupar uma dessas cadeiras restam 8 vazias pra
>> distribuir entre as pessoas, mas entre duas pessoas deve ter ao menos
>> cadeira vazia, então -> 9!/5!x4!=136
>> No terceiro caso temos 13 cadeiras pra colocar 5 pessoas, logo 8 vazias,
>> mas entre duas pessoas devemos ter uma cadeira vazia pelo menos uma vazia
>> -> 9!/4!x5!=136
>> Total-> (2x136+136)x5!=45360
>>
>> On Dec 10, 2015, at 16:45, Vanderlei Nemitz <vanderma...@gmail.com>
>> wrote:
>>
>> Pessoal, gostaria de uma ajuda com essa questão. Vi em um site a
>> resposta 45360, mas não concordo. Encontrei um valor bem menor. Obrigado!
>>
>> Vanderlei
>>
>> *Cinco pessoas devem se sentar em 15 cadeiras colocadas em torno de uma
>> mesa circular. De quantos modos isso pode ser feito se não deve haver
>> ocupação simultânea de duas cadeiras adjacentes? *
>>
>> --
>> Esta mensagem foi verificada pelo sistema de antivírus e
>> acredita-se estar livre de perigo.
>>
>>
>> --
>> Esta mensagem foi verificada pelo sistema de antivírus e
>> acredita-se estar livre de perigo.
>>
>
>
> --
> Esta mensagem foi verificada pelo sistema de antivírus e
> acredita-se estar livre de perigo.
>
>
> --
> Esta mensagem foi verificada pelo sistema de antivírus e
> acredita-se estar livre de perigo.
>

-- 
Esta mensagem foi verificada pelo sistema de antiv�rus e
 acredita-se estar livre de perigo.



  1   2   >